N204 Practice Quizes, Fundamentals Quiz, Health and Physical Assessment, Leadership EAQ's, EAQ NCLEX, Maternity Chap 28, Maternity and Women's Health Nursing - Newborn, Nur 106- Module G2, Pediatric Growth & Development EAQ, Nursing Sciences EAQ, The...

Ace your homework & exams now with Quizwiz!

Which stage of psychosocial development is observed in a 5-year-old child according to Erikson's theory? 1 Initiative vs. guilt 2 Trust vs. mistrust 3 Industry vs. inferiority 4 Autonomy vs. shame and doubt

1 Initiative vs. guilt Children between 3 and 6 years of age like to pretend and try out new things. Conflicts may often occur between the child's desire to explore and the limits placed on his or her behavior, which may lead to frustration and guilt. Therefore, the initiative vs. guilt stage is seen in 3- to 5-year-old children. The trust vs. mistrust stage is observed in infancy (birth to 18 months). The industry vs. inferiority stage is seen in children between 6 and 12 years of age. The autonomy vs. shame and doubt stage is seen in children between 18 months and 3 years of age.

Which statement by the parents of an infant with Erb palsy indicates to the nurse that they have an accurate understanding of their infant's prognosis? 1 "Complete recovery takes a few months." 2 "This is a progressive paralysis with no cure." 3 "Physical therapy will be necessary for a year." 4 "Surgery will be needed to correct the problem."

1 "Complete recovery takes a few months." The nerves that have been stretched take about 3 months to recover from the trauma sustained during the birth process. The paralysis is not progressive, and the prognosis is usually excellent. Passive range of motion and intermittent splinting performed by a family member are generally all the interventions that are necessary; only in rare instances, when avulsion of the nerves results in permanent damage, is orthopedic or surgical intervention necessary.

Which immunoglobulins (Ig) are transferred from a mother to a fetus? 1 IgG 2 IgA 3 IgD 4 IgM

1 IgG Significant amounts of maternal IgG antibodies are passed on to a newborn and confer immunity against antigens. IgA is not present at birth but is found in saliva and tears by 2 to 5 weeks of age. The production of IgD is gradual, and increases progressively during childhood. Significant amounts of IgM are produced at birth, and adult levels are reached by 9 months of age.

Which would the nurse document for a client with drooping of the eyelid over the pupil? 1. Ptosis 2. Ectropion 3. Entropion 4. Nystagmus

1. Ptosis * Eyelids that droop over the pupil is called ptosis, which is how the nurse would document the finding. Ectropion is when the eyelid margins turn out. Entropion is similar, but is when the eyelid margins turn in and sometimes can cause irrigation of the conjunctiva and cornea. Nystagmus is an involuntary oscillation of the eye, and usually occurs after an eye injury.

What should the nurse teach parents about their newborn's diagnosis of phenylketonuria (PKU)? 1. A low-phenylalanine diet is required. 2. Phenylalanine is not necessary for growth. 3. Phenylalanine can be administered to correct the deficiency. 4. A substitute for phenylalanine is an increased amount of other amino acids.

1.A low-phenylalanine diet is requiredReducing dietary phenylalanine helps prevent brain damage. The PKU diet is planned to maintain the serum phenylalanine level at 2 to 8 mg/100 mL. Phenylalanine is essential for growth and development of the brain. Administering phenylalanine is contraindicated. There are no substitute for phenylalanine, which is one of the essential amino acids.

Which technique would the nurse suggest to a laboring woman's partner that involves gently stroking the woman's abdomen in rhythm with her breathing during a contraction? 1 Massage 2 Effleurage 3 Acupressure 4 Counterpressure

2

Preterm infant Small-for-gestational-age infant Large-for-gestational-age infant

A nurse is testing a newborn's heel blood for the level of glucose. Which newborn does the nurse anticipate will experience hypoglycemia? (Select all that apply.)

"We'll have to have the baby fitted with prosthetic devices before he'll be able to walk."

A nurse reviews the prescribed treatment with the parents of an infant born with bilateral clubfeet. Which parental statement indicates to the nurse that further education is required?

Continue to monitor the blood glucose level per policy.

A nurse who is monitoring the blood glucose level of the term infant of a diabetic mother (IDM) identifies a blood glucose level of 48 mg/dL. What should the nurse do?

A nursing student is taking down notes about paradigm. Which point noted down by the nursing student needs correction?

A paradigm is the perspective of a profession.

Which patients are at risk of developing health care-associated infections (HAIs)? Select all that apply.

A patient with laryngeal cancer A patient with diabetes mellitus A patient with an indwelling urinary catheter

Which program is an example of a continuing education program?

A program on caring for the elderly with dementia offered by a university

antivenin

A serum that counteracts the effect of venom from an animal or insect.

A client with a coronary occlusion is experiencing chest pain and distress. Which is the primary reason that the nurse administers oxygen? 1 To prevent dyspnea 2 To prevent cyanosis 3 To increase oxygen concentration to heart cells 4 To increase oxygen tension in the circulating blood

3

A client in the emergency department states, "I was bitten by a raccoon while I was fixing a water pipe in the crawl space of my basement." Which is the most effective first-aid treatment for the nurse to use for this client?

Cleansing the wound with soap and water

A nurse is evaluating the effectiveness of treatment for a client with excessive fluid volume. What clinical finding indicates that treatment has been successful?

Clear breath sounds

Which nursing model includes a registered nurse (RN) paired with technical assistance?

Co-primary nursing model

Clients that can delay treatment for more than 3 hours and remain stable

Green tag. Yellow tag 30min-2hr

The school nurse conducts a class in nutrition planning for parents. What is the goal of school health nursing programs?

Health promotion

2 Tachycardia

Heart rate over a 100

A nurse is preparing a community health program for senior citizens. The nurse teaches the group that what physical findings are typical in older adults?

Increased blood pressure and decreased hormone production

For which client(s) should the nurse consider family members as the primary source of information? Select all that apply.

Infant or child During traumatic emergency When critically ill, disoriented

The registered nurse (RN) administers intravenous fluids to a client who was in a motorcycle accident. Which assessments made by the nurse would be appropriate based on the principle of right task of delegation?

Institutional policies of drug administration

Which internal variable influences health beliefs and practices?

Intellectual background

A nurse finds that there is an inaccurate match between clinical cues and the nursing diagnosis. What is the category of the diagnostic error?

Interpreting

A nurse is counseling a client who has gonorrhea. What additional fact about gonorrhea, besides the fact that it is highly infectious, should the nurse teach this client?

It can produce sterility.

What makes a crisis access hospital (CAH) different from an intensive care unit (ICU)?

It provides temporary care for 96 hours or less.

3 examples of subjective data

Pain, Dizziness, Exhaustion

A nurse takes into consideration that the key factor in accurately assessing how a client will cope with body image changes is what?

Perception of the change

Clinical judgment

a judgment based on experience in observing and treating patients

The nurse is assessing four infants. Which infant does the nurse anticipate to be of abnormal weight?

age = 5 months weight at birth = 3.3 kg current weight = 8.5 kg

A nurse is preparing to administer an intravenous piggyback medication to a client who is receiving a continuous infusion of intravenous (IV) fluids. What is the priority nursing intervention?

check the compatibility of the medication and the continuous IV solution.

A child is brought to the pediatric clinic because he has been vomiting for the past 2 days. What acid-base imbalance would the nurse expect to occur from this persistent vomiting? a. Hyperkalemia b. Hypernatremia c. Acidosis d. Alkalosis

d

Palliative care

hospice care; taking care of the whole person—body, mind, spirit, heart and soul—with the goal of giving patients with life-threatening illnesses the best quality of life they can have through the aggressive management of symptoms

Who is responsible for client outcomes in the functional model of care?

nurse manager

Which clients suffer from impaired near vision? Select all that apply.

presbyopia & hyperopia

5 rights of delegation

right task, right circumstances, right person, right direction/communication, and right supervision/evaluation.

What is the correct order of phases a client experiences in the event of a change in body image following an illness?

shock, withdrawal, acknowledgement, acceptance & rehabilitation

Hypoglycemia

A small-for-gestational-age (SGA) newborn who has just been admitted to the nursery has a high-pitched cry, appears jittery, and exhibits irregular respirations. What complication does the nurse suspect?

Which degree of edema will result in a 6-mm deep indentation upon pressure application?

3 +

They have a tendency to collapse with each breath.

What does the nurse expect concerning the alveoli in the lungs of a 28-week-gestation neonate?

ABCS

airway, breathing, circulation

dysphagia

condition in which swallowing is difficult or painful

Etiology

the study of the causes of diseases

Protecting the sac with moist sterile gauze

After an uneventful pregnancy a client gives birth to an infant with a meningocele. The neonate has 1-minute and 5-minute Apgar scores of 9 and 10, respectively. What is the priority nursing care for this newborn?

Expected movements and behaviors

What should the nurse's initial discussion include to best help new parents understand the unique characteristics of a newborn?

Avoid squeezing them and don't try to wash them off.

When a nurse brings a newborn to a mother, the mother comments about the milia on her infant's face. What information should the nurse include when responding?

A mother reports that her child has been scratching the anal area and complaining of itching. What does the nurse suspect based on this information? a. Pinworms b. Giardiasis c. Ringworm d. Roundworm

a

Who is the designated delegator?

Nurse manager

A nurse needs to provide preventive and primary care to adults during a health camp. Which of these actions should the nurse perform? Select all that apply.

The nurse should discuss vaccinations. The nurse should discuss family planning The nurse should instruct the health camp about road safety measures.

What is the ultimate goal of delegation?

To maximize patient care outcomes

What type of approach would be a comforting approach that communicates concern and support?

Touch

Which site is best used to inspect a client who is suspected to have jaundice?

Sclera

Which component of delegation is retained while the delegator is delegating the client's care task to the nursing aide?

Accountability

Nursing Process

Assessment Diagnosis Planning Implementation Evaluation

Which group benefits from Medicare?

People who are 65 years or older

Which work is automatically increased for the delegator when there is a decrease in direct client care?

Supervision

mucositis

painful inflammation of oral mucous membranes

hematuria

presence of blood in the urine

RACE -

Rescue, Activate alarm, Confine the fire, Evacuate/Extinguish

Numerous superficial veins

Which sign indicates to the nurse that a neonate is preterm?

pyloric stenosis.

narrowing of the opening of the stomach to the duodenum

Reflex irritability: cry Respiratory rate: good cry Heart rate: 110 beats/min

After a difficult birth, a neonate has an Apgar score of 8 after 5 minutes. Which signs met the criteria of 2 points? (Select all that apply.)

The nurse is developing a nursing diagnosis for a client after surgery. The nurse documents the "related to" factor as first time surgery. Which assessment activity enabled the nurse to derive this conclusion?

The nurse asks the client to explain the surgery.

A client who has sustained an accident says, "I have a dream of conquering the world's highest mountain range." To which level of need does the given scenario refer to, according to Maslow's hierarchy of needs?

Toddler Preschool

Voids six or more times a day

The nurse assures a breastfeeding mother that one way she will know that her infant is getting an adequate supply of breast milk is if the infant gains weight. What behavior does the infant exhibit if an adequate amount of milk is being ingested?

Which intervention by the home health care nurse conforms to the use of safety competency while providing health care?

The nurse ensures the furniture does not obstruct the client's movement.

A nurse is working in a health care organization that has Magnet status. What specific responsibility does the nurse have in this organization?

The nurse must collect data for comparison against a national level.

Opiate withdrawal syndrome

The nurse observes that 12 hours after birth the neonate is hyperactive and jittery, sneezes frequently, has a high-pitched cry, and is having difficulty suckling. Further assessment reveals increased deep tendon reflexes and a diminished Moro reflex. What problem does the nurse suspect?

Buccal smear

The parents of a newborn are told that their neonate may have Down syndrome and that additional diagnostic studies will be done to confirm this diagnosis. What procedure does the nurse expect to be performed?

A client's sputum smears for acid-fast bacilli (AFB) are positive, and transmission-based airborne precautions are prescribed. What should the nurse teach visitors to do?

Wear a particulate respirator mask

Which statement by a mother may indicate a cause of her sons vitamin C deficiency? a. We get our fruits from homemade preserves. b. We use milk from our own goats. c. We grow all our own vegetables. d. Were not big meat eaters

a

The nurse is interviewing parents of an infant with pyloric stenosis. What would the nurse expect the parents to report? a. Diarrhea b. Projectile vomiting c. Poor appetite d. Constipation

b

What interventions will the nurse perform when feeding a child with pyloric stenosis? (Select all that apply.) a. Give a formula thinned with water. b. Burp the infant before and during feeding. c. Give the feeding slowly. d. Refeed if the infant vomits. e. Position infant on left side after feeding.

b, c, d

A child is brought into the ED with suspected appendicitis. What signs and symptoms does the nurse expect to assess? (Select all that apply.) a. Left lower quandrant pain b. Guarding c. Rebound tenderness d. Decreased C-reactive protein e. Pain on lifting thigh when supine

b, c, e

What does the nurse expect the appearance of the stools of a child with celiac disease to be? a. Ribbon like b. Hard, constipated c. Bulky, frothy d. Loose, foul-smelling

c

An infant is admitted to the hospital with severe isotonic dehydration. For what is this child at the highest risk? a. Metabolic alkalosis b. Hypocalcemia c. Sepsis d. Shock

d

glomerulonephritis

inflammation of the glomeruli of the kidney

Objective data

information that is seen, heard, felt, or smelled by an observer; signs

Arrange the hierarchy of needs in ascending order beginning with the highest priority needs as defined by Maslow.

physiological, safety and security, love and belonging, self-esteem and self-actualization

Which pain scale would the nurse use to measure the intensity of pain in toddlers? 1 FACES scale 2 Visual analog scale 3 Numerical rating scale 4 Verbal descriptor scale

1

The nurse is assessing a client who underwent abdominal surgery 10 day ago. The client complains of pain in the abdomen. Which type of pain would the client experience. 1. Visceral pain 2. Somatic pain 3. Referred pain 4. Intractable pain

1. Visceral pain * Visceral pain arises from visceral organs such as the pancreas, which results from the stimulation of pain receptors in the abdominal cavity. Somatic pain arises from bone, joint, muscle, skin, or connective tissue and is usually aching or throbbing in quality and well localized. Referred pain is experienced in clients with tumors, in which pain is felt in a part of the body other then its actual source. Intractable pain is a neuropathic pain that is severe, constant pain that is not curable.

What should the nurse do when the defining characteristics of assessment data for a client can apply to more than one diagnosis? Select all that apply. 1 Reassess the client. 2 Reject all diagnoses. 3 Gather more information. 4 Identify related factors. 5Review all defining characteristics.

3 Gather more information. 4 Identify related factors. 5Review all defining characteristics

Getting an informed consent signed by the mother of the baby

A 15-year-old emancipated minor gave birth to a boy 36 hours ago and has requested a circumcision. What is the nurse's priority?

Deafness Cardiac anomalies

A nurse suspects that a newborn's mother had rubella during the first trimester of pregnancy. Which newborn problems support this assumption? (Select all that apply.)

A client with multiple myeloma who is receiving chemotherapy has a temperature of 102.2° F. The temperature was 99.2° F when it was taken six hours ago. A priority nursing intervention is to:

Administer the prescribed antipyretic and notify the charge nurse or primary health care provider

A nurse manager transfers the task of caring for a client who has undergone appendectomy to a registered nurse (RN). Which element of the healthcare system is the RN practicing?

Assignment

3 examples of objective data

Blood pressure Heart rate Respirations

Which is the priority nursing action to include in a disaster plan for the radioactive dust and smoke that can cause illness from a radiologic dispersal device (RDD)?

Covering the nose

A client is diagnosed with gastroenteritis. What does the nurse determine is the basic intention underlying the unique dietary management for this client?

Eliminate chemical, mechanical, and thermal irritation.

Before the nurse can be an advocate for a client who is homosexual who has acquired immunodeficiency syndrome (AIDS), the nurse needs to do what?

Identify personal attitudes and feelings about homosexuality

A hospital organization plans to conduct a study on the effect of dried plums for lowering the risk of colon cancer. After selecting the subjects, a nurse researcher provides adequate information about the research and then inquires about the preference of the subjects to associate with the research. What does this procedure indicate?

Informed consent

preferred provider organizations (PPOs)

Preferred provider organizations reimburse nursing home payments. Preferred provider organizations cover children who are not poor enough for Medicaid. Preferred provider organizations have deductibles that clients must meet before the insurance pays.

Who is accountable for the ongoing evaluation of a client's care?

Registered Nurse (RN)

Which task is achieved by the delegator when he or she engages in self-care to enhance his or her ability to care for the healthcare team?

Renewing

A client is admitted with the diagnosis of tetanus. For which clinical indicators should the nurse assess the client? (Select all that apply.

Restlessness Muscular rigidity Respiratory tract spasms Spastic voluntary muscle contractions

A client is to receive an intravenous (IV) antibiotic in 50 mL of 0.9% sodium chloride to be administered over 20 minutes. At what rate should the nurse set the infusion pump? Record your answer using a whole number. __________ mL/hr

150 ml/hr

Which critical thinking skill is being used when the nurse applies knowledge and experience to client care? 1. Analysis 2. Evaluation 3. Explanation 4. Interpretation

3. Explanation * When the nurse is using experience to care for clients, the critical thinking skill of explanation is being applied. Analysis is applicable when the information is collected with an open mind. Evaluation is applicable when the information is used to determine nursing actions. Interpretation is involved when orderly data is collected.

Which physical skin finding indicated opioid abuse? 1. Diaphoresis 2. Red, dry skin 3. Needle marks 4. Spider angiomas

3. Needle marks * Needle marks of the skin indicate opioid abuse. Diaphoresis indicated sedative hypnotic abuse. Red, dry skin indicates phencyclidine abuse. Spider angiomas indicate alcohol abuse.

After abdominal surgery, a client reports pain. Which action would the nurse take first? 1 Reposition the client. 2 Obtain the client's vital signs. 3 Administer the prescribed analgesic. 4 Determine the characteristics of the pain.

4

Which content type of play allows a child to experience pleasure by swinging? 1 Skill play 2 Pretend play 3 Social-affective play 4 Sense-pleasure play

4 Sense-pleasure play Sense-pleasure play is a nonsocial stimulating experience in which the pleasurable experiences are derived from the environment, handling of raw materials, and body motion such as swinging, bouncing, and rocking. In skill play, infants persistently demonstrate and exercise their newly acquired abilities. The simple, imitative, dramatic play of toddlers, such as using a telephone, driving a car, or rocking a doll is called pretend play or dramatic play. In social-affective play, the infant takes pleasure in relationships with people. As adults talk, touch, nuzzle, and in various ways elicit responses from an infant, the infant soon learns to provoke parental emotions and responses.

Promote clotting of the blood

A 7-lb newborn is admitted to the nursery with a prescription for intramuscular phytonadione (vitamin K, Aquamephyton) 1 mg. The nurse explains to the parents that this vitamin is administered to:

The ribcage is not compressed, then released during birth.

A client has a cesarean birth. The nurse monitors the newborn's respiration because infants subjected to cesarean birth are more prone to atelectasis. Why does this occur?

ABO incompatibility

A client who has type O Rh-positive blood gives birth. The neonate has type B Rh-negative blood. Eleven hours after birth, the infant's skin appears yellow. What is the most likely cause?

Which of these clients can be provided care safely by unlicensed nursing personnel? Select all that apply.

A client who is stable A client who is recovering

The nurse is reviewing blood screening tests of the immune system of a client with acquired immunodeficiency syndrome (AIDS). The nurse expects to find:

A decrease in CD4 T cells

Color

A nurse assesses a healthy 8-lb 8-oz (3860-gm) newborn who was given Apgar scores of 9 at 1 minute and 10 at 5 minutes. Which category of the Apgar score received a 1 rating at one minute?

Which intervention by the nurse is an important aspect of client-centered care according to the survey conducted by the Picker Institute?

Asking the client if the family should be involved in his or her care

What are the similarities between Medicare and Medicaid? Select all that apply.

Both programs assist older clients. Both programs pay for home care services

Which employee should be competent in critical thinking, leadership, communication, and time management skills? Select all that apply.

Charge nurse, registered nurse

Who would the registered nurse state is accountable for establishing systems to communicate competency requirements related to delegation?

Chief Nursing officer (CNO)

A client with severe burns and injuries from a bomb blast is admitted in the emergency unit. Which triaging priority is preferable for the client?

Class I

These findings indicate that the infant may have a pneumothorax, and the health care provider should be contacted immediately.

Continuous positive-pressure ventilation therapy by way of an endotracheal tube is started in a newborn with respiratory distress syndrome (RDS). The nurse determines that the infant's breath sounds on the right side are diminished and that the point of maximum impulse (PMI) of the heartbeat is in the left axillary line. How should the nurse interpret these data?

Which characteristic should the nurse use during the nursing process as a guide for delegation?

Critical analysis

According to Watson's theory, in which practice is the nurse least likely to engage?

Curing the disease before engaging in care of the client

A nurse is caring for a client with a diagnosis of acute salpingitis. Which condition most commonly causing inflammation of the fallopian tubes should the nurse include when planning a teaching program for this client?

Gonorrhea

Which would the nurse discuss with new parents to assist them in preparing for infant care?

Learning specific behaviors involving states of wakefulness to promote positive interactions

While caring for a group of clients from different ethnicities, the nurse observes that a client from Ireland is stoic and not complaining about pain. Which theory should the nurse follow in this situation?

Leininger's Theory

The registered nurse is organizing a community health care program for administering tetanus vaccinations. Which member of the health care team is most suitable for delegating the task of administering vaccinations

Licensed practical nurse

A client with foot ulcers is admitted to the hospital. The nurse manager should delegate the task of maintaining hygiene to which staff members to maximize efficient use of human resources? Select all that apply.

Patient care associate (PCA) Unlicensed nursing practitioner (UNP)

Which of these is a part of health belief model?

Perception of susceptibility to an illness

What is the emergency care for a drowning client?

Safely rescuing the patient

A group of clients who were in a bus accident is admitted to the emergency department with injuries. Which group is considered urgent according to the three-tier triage system?

Severe abdominal pain

What is the main motto of the Hersey's 2006 model?

Situational leadership

What information should the nurse provide for a client who is discharged from the health care facility with a surgical wound? Select all that apply.

Skill to care for the surgical wound Safe and effective use of medications List of appropriate community resources

SOP

Standard Operating Procedure (defined in Queensland mining legislation)

A middle-aged client says, "I have been unsuccessful in raising my kids." Which stage should the nurse expect the client to have reached, according to Erikson's theory of psychosocial development?

The client has reached the Generativity versus Self-Absorption and Stagnation stage.

Which feature is characteristic of a risk nursing diagnosis?

The diagnosis does not have related factors.

What is the incubation period for an infectious disease?

The interval between entrance of pathogen into body and appearance of first symptoms

A client arrives at the clinic after being bitten by a raccoon in an area in the woods where rabies is endemic. When considering the client's needs, the nurse recalls that rabies is a:

Viral infection characterized by convulsions and difficulty swallowing

A nurse is assessing a client who underwent abdominal surgery 10 days ago. The client complains of pain in the abdomen. What type of pain does the client experience?

Visceral pain

Mucous membranes of the mouth

Where is the best area for the nurse to determine adequate tissue oxygenation in a neonate born of black parents?

A nursing student is listing various stages of Lawrence Kohlberg's Theory of Moral Development. Which situation indicates that the individual has reached the Instrumental Relativist Orientation stage?

"An individual identifies that there is more than one right view."

A nursing instructor asks a student to describe Betty Neuman's theory. Which statement by the student indicates the need for further education?

"Betty Neuman's theory outlines that the external environmental factors act as stressor."

Isoniazid (INH) is prescribed as a prophylactic measure for a client whose spouse has active tuberculosis (TB). What statements by the client indicate that there is a need for further teaching? (Select all that apply.)

"I sometimes allow our children to sleep in our bed at night." "I know I also have tuberculosis because the skin test was positive." "I'll be skipping the wine but enjoying the cheese at my neighbor's party.

Isoniazid (INH) is prescribed as a prophylactic measure for a client whose spouse has active tuberculosis (TB). What statements by the client indicate that there is a need for further teaching? (Select all that apply.)

"I sometimes allow our children to sleep in our bed at night." "I know I also have tuberculosis because the skin test was positive." "I'll be skipping the wine but enjoying the cheese at my neighbor's party.

A young client with schizophrenia says, "I'm starting to hear voices." What is the nurse's most therapeutic response? "How do you feel about the voices, and what do they mean to you?" "You're the only one hearing the voices. Are you sure you hear them?" "The health team members will observe your behavior. We won't leave you alone." "I understand that you're hearing voices talking to you and that the voices are very real to you. What are the voices saying to you?"

"I understand that you're hearing voices talking to you and that the voices are very real to you. What are the voices saying to you? Acknowledging that client is hearing voices and that the voices are very real to the client validates the presence of the client's hallucinations without agreeing with them, which communicates acceptance and can form a foundation for trust; it may help the client return to reality. The nurse also needs to assess the content of the voices to determine the risk of self-injury or violence against others. The client's contact with reality is too tenuous to explore what the voices mean. Saying that the client is the only one hearing the voices and asking whether the client is sure the voices are being heard demeans the client, which blocks the development of a trusting relationship and future communication. Telling the client that the health team members will observe the behavior and that the client won't be left alone is condescending and may impair future communication

A nurse is recollecting Sigmund Freud's psychoanalytical model of personality development. What are the characteristics of the phallic stage, as per this model? Select all that apply.

"In this stage, a child may develop an oedipal complex." "In this stage, a girl may experience 'penis envy' feelings." "In this stage, the genital organs are the focus of pleasure.

A registered nurse is teaching a student nurse about a rapid-improvement event (RIE), a quality improvement model. Which statement by the student nurse indicates effective learning?

"It is a very intense, usually week-long, event."

A nurse conducting a research study is obtaining informed consent from a research subject. Which statement made by the nurse to the client needs correction?

"Let me explain all the details about the research and why it is the best available method of treatment."

A school-aged child is brought to the clinic by the mother, who states, "Something is very wrong. My child never seems happy and refuses to play." When assessing this child for depressed behavior, what statement should the nurse initially begin with? "Tell me about yourself." "Let's talk about what you do after school." "Can you tell me what's making you so unhappy?" "Why does your mother think that you're unhappy?"

"Let's talk about what you do after school." A structured but nonthreatening question such as asking what the child does after school avoids beginning with the problem and may put the child at some ease, producing information that may be useful. The statement "Tell me about yourself" is too open and global; the child will probably not know how to answer this question or know where to begin. The child may not know the answer to the question "Can you tell me what's making you so unhappy?" Asking "Why does your mother think that you're unhappy?" will probably produce an "I don't know" response; the focus should be on the child, not the mother.

A registered nurse is teaching a nursing student about Nightingale's theory, which is an initial model for nursing. Which statements of a nursing student indicate an understanding of the theory? Select all that apply.

"Nightingale's theory deals with visionary principles that include areas of practice, research, and education." "Nightingale's theory deals with descriptive theories that provides nurses with a way to think about clients and their environment." "Nightingale's theory focuses on the fact that nursing is caring through the environment and helping the client deal with symptoms related to illness."

A registered nurse is teaching a student nurse about Gesell's theory of biophysical development. Which statement by the student nurse indicates the need for further teaching? Select all that apply.

"Proximodistal growth describes the sequence in which growth is fastest from the head down." "In the cephalocaudal pattern, growth starts at the center of the body and moves toward the extremities."

The registered nurse is evaluating the statements made by a student nurse after teaching ways to make appropriate delegation decisions. Which statement made by the student nurse indicates a need for correction?

"The delegator should recheck and redo the work of the delegatee."

Ampicillin 250 mg by mouth every six hours is prescribed for a client who is to be discharged. Which statement indicates to the nurse that the client understands the teaching about ampicillin?

"The medicine should be taken one hour before or two hours after meals."

A client with tuberculosis asks the nurse about the communicability of the disease. Which is the best response by the nurse?

"Untreated active tuberculosis is communicable."

A nurse is planning to provide discharge teaching to the family of a client with acquired immunodeficiency syndrome (AIDS). Which statement should the nurse include in the teaching plan?

"Wash used dishes in hot, soapy water."

A client presents to the healthcare facility with abdominal pain. Which question should the nurse ask the client to obtain information about concomitant symptoms?

"What other discomfort do you experience?"

A client with schizophrenia who has auditory hallucinations is withdrawn and apathetic. What should the nurse say to involve this client in an activity? "You'll get a reward if you go to the gym." "Would you like to participate in the group walk today?" "Those voices you hear would like it if you did a little exercise." "There's a positive relationship between exercise and good mental health."

"Would you like to participate in the group walk today?" is a declarative statement that invites the client to walk, and the client can comply without making a verbal decision. A client with schizophrenia is often ambivalent, rendering decision-making difficult. A withdrawn, apathetic client probably will not internalize or appreciate rationales for interventions. Saying that the voices want the client to exercise supports the client's hallucinations

The registered nurse (RN) and unlicensed assistive personnel (UAP) have been working together for two years. Which statement made by the RN would be appropriate after delegating a task to the UAP?

"You know what to do and when to report"

A disturbed client says, "The voices are saying that I killed my husband." What is the best response by the nurse? "I just saw your husband, and he's doing fine." "Tell me more about your concerns for your husband." "We'll put you in a private room where you'll be safe." "You seem to be having very frightening thoughts right now."

"You seem to be having very frightening thoughts right now" demonstrates that the nurse understands the client's feelings; reflection opens a channel for communication. The nurse cannot talk the client out of her delusions by pointing out reality. Focusing on delusional content only reinforces false beliefs. "We'll put you in a private room where you'll be safe" does not reflect the content of the client's statement.

While receiving a preoperative enema, a client starts to cry and says, "I'm sorry you have to do this messy thing for me." What is the nurse's best response? "I don't mind it." "You seem upset." "This is part of my job." "Nurses get used to this."

"You seem upset." The nurse should identify clues to a client's anxiety and encourage verbalization of feelings. Saying it is part of the job focuses on the task rather than on the client's feelings. Saying "I don't mind it" or "Nurses get used to this" negates the client's feelings and presents a negative connotation.

Which statement by the parent of a 9-month-old supports the nurse's conclusion that the infant displays developmental accomplishments appropriate for this age? 1 "My child can throw a ball." 2 "My child can crawl backwards." 3 "My child can locate small objects." 4 "My child shows jerky movements upon hearing loud sounds."

1 "My child can throw a ball." A child who is 8 to 12 months old is capable of throwing objects. Therefore, this statement made by the parent indicates that the 9-month-old infant has achieved the developmental milestone appropriate for the age. An infant who is 4 to 7 months old is capable of crawling backwards and can locate small objects. These actions can be observed in an infant who is 9 months old, but it does not indicate major developmental accomplishments as per its age. Showing jerky movements upon hearing loud noise indicates that the child is showing startle reflex, which is an involuntary reflex seen from the birth to 4 months of age.

At which age does an infant have a strong grasp reflex? 1 1 month 2 2 months 3 3 months 4 4 months

1 1 month The grasp reflex is strong in 1-month-old infants. Fading of the grasp reflex is observed at 2 months of age. The absence of a grasp reflex is observed in a 3-month-old infant. A 4-month-old infant grasps objects with both hands. However, a grasp reflex is absent in this infant.

Which reading of a newborn's pulse may indicate an abnormality in the function of the cardiovascular system? 1 100 beats/min 2 120 beats/min 3 130 beats/min 4 140 beats/min

1 100 beats/min The normal pulse rate of a newborn ranges from 120 to 160 beats/min. Therefore, a pulse rate of 100 beats/min may indicate an abnormality. Pulse rates of 120 beats/min, 130 beats/min, and 140 beats/min are within normal range.

Which type of cerebral palsy may cause a wide-based gait in children? 1 Ataxic 2 Spastic 3 Dyskinetic 4 Mixed type

1 Ataxic Ataxic cerebral palsy is caused by damage to the cerebellum, which is essential for the coordination of muscle movements and balance. Therefore, a wide-based gait is observed in patients with ataxic cerebral palsy. Spastic cerebral palsy causes hypertonicity with poor control of posture, balance, and coordinated motion. Dyskinetic cerebral palsy is characterized by athetoid and dystonic movements. Mixed cerebral palsy is a combination of spastic and dyskinetic cerebral palsy, and symptoms of both conditions are present.

Which behavior is observed in a newborn in deep sleep? 1 Closed eyes 2 Irregular breathing 3 Occasional smiling 4 Rapid eye movements

1 Closed eyes A newborn in deep sleep will have closed eyes, regular breathing, and no eye movements. A newborn in light sleep would have irregular breathing, rapid eye movements, and may smile.

Which condition results from an abnormal organization of cells into a particular tissue type? 1 Dysplasia 2 Disruption 3 Deformation 4 Malformation

1 Dysplasia Dysplasias result from abnormal organizations of cells into a particular tissue. Disruptions result from the breakdown of normal tissue. Extrinsic mechanical forces on previously normal tissue cause deformations. A malformation results when developmental processes lead to an abnormally formed body part or organ.

An infant with a myelomeningocele is scheduled for surgery to close the defect. Which nursing action best facilitates the parent-child relationship in the preoperative period? 1 Encouraging the parents to stroke their infant 2 Allowing the parents to hold their infant in their arms 3 Referring the parents to the Spina Bifida Association of America 4 Teaching the parents to use special techniques when feeding the infant

1 Encouraging the parents to stroke their infant Because the infant cannot be held, tactile stimulation helps meet the infant's needs and fosters bonding with the parents. An infant with an unrepaired myelomeningocele cannot be held in the arms. Referrals will be more appropriate at a later time. Although special feeding techniques are important in the postoperative period, they may not improve the parent-infant relationship.

The parents of a toddler who has been admitted to the pediatric unit for surgery to correct hypospadias ask the nurse when this defect happened. The nurse responds that it usually occurs during fetal development, in the: 1 First 12 weeks 2 Third trimester 3 Second 16 weeks 4 Implantation phase

1 First 12 weeks The critical period of organogenesis occurs during the first trimester, when fetal development is most likely to be adversely affected. The fetus is less vulnerable after the first trimester because organ development is complete. The fetus is less vulnerable to major anomalies during the second 16 weeks because all major organ systems already are formed. At the time of implantation cellular differentiation has not occurred; the genital bud appears in the seventh week.

Which theory explains psychosexual development through infancy to adolescence? 1 Freud's theory 2 Piaget's theory 3 Erikson's theory 4 Kohlberg's theory

1 Freud's theory Psychosexual development through the five developmental stages is explained by Freud's theory. Piaget's theory explains cognitive and moral development from infancy to adolescence. Erikson's theory explains psychosocial development. Kohlberg's theory demonstrates the development of moral reasoning.

Which gross motor skill can be observed in a 7-month-old infant? 1 Sits alone without support 2 Creeps on hands and knees 3 Walks holding onto furniture 4 Sits down from a standing position

1 Sits alone without support Infants between 6 and 8 months of age can sit alone without support. Infants can creep on their hands and knees at the age of 8 to 10 months. Infants who are 10 to 12 months of age can walk while holding onto furniture and can sit down from a standing position due to their well-developed motor skills.

After interacting with a preschooler, the nurse concludes that the child has normal development according to Fowler's spiritual development. Which behavior of the child supports the nurse's conclusion? 1 The child imitates the adults as they pray. 2 The child does not exhibit any spiritual behavior. 3 The child accepts the existence of a supreme power. 4 The child questions the religious practice and its benefits.

1 The child imitates the adults as they pray. A preschooler is in the intuitive-projective stage of Fowler's spiritual development, and would imitate the others as they pray and perform other religious activities. An infant is in the undifferential stage of Fowler's development and will not have any spiritual behavior. The school-age child is in mythical-literal stage of Fowler's spiritual development, and may accept the existence of a supreme power. An adolescent is in the synthetic-convention stage of Fowler's spiritual development, and may question religious practice and its benefits.

The nurse is evaluating the dietary plan of a 6-month-old infant. Which action by the infant's mother needs correction? 1 The mother gives low-fat milk to the infant in a bottle. 2 The mother gives teething crackers to the infant for pain relief. 3 The mother refrains from giving iron supplements to the infant. 4 The mother refrains from giving chopped table food to the infant.

1 The mother gives low-fat milk to the infant in a bottle. Cholesterol is required for proper neurological development in infants. Therefore, low-fat milk should not be given to infants and toddlers. Finger foods, such as teething crackers, should be introduced at the age of 6 months, as it helps provide complete nutrition to the infant. The nurse should encourage the mother to give whole cereals instead of iron supplements to the child. Chopped table food can be given to the child at the age of 9 months.

What could be a cause of a stiff, tense appearance of the sutures in a newborn? 1 The neonate is crying. 2 The neonate is sneezing. 3 The skull bones of the neonate are fused. 4 The neonate has irregular breathing pattern.

1 The neonate is crying. Sutures are the layers of connective tissue formed between the bones of the cranium. They facilitate the molding of the cranium during vaginal delivery. Conditions like crying and coughing may cause a temporary bulging in the sutures, which may cause stiffness and tension on palpation. Sneezing and an irregular breathing pattern may not directly influence the stiffness of the sutures. The fusion of skull bones is not generally observed in newborns except in some rare conditions.

Which education would the nurse provide the parents of an infant with pyloric stenosis?

1 It is unlikely that surgery will be necessary. Correct 2 This is a condition with an excellent prognosis. 3 This condition results from an error of metabolism. Incorrect 4 Special feedings will be needed for a few weeks after surgery. answer is 2

Which therapeutic communication technique is useful when the nurse and a client have a conversation and the client begins to repeat the conversation to himself or herself? 1 Focusing 2 Clarifying 3 Paraphrasing 4 Summarizing

1 focusing

A 4-year-old child is admitted to the pediatric neurological service with a seizure disorder. Shortly after admission, while in bed, the child has a generalized seizure. What nursing actions are most appropriate? Select all that apply. 1. Assessing the seizure 2.Taking the child's vital signs 3.Turning the child on the side 4.Pulling the padded side rails up 5.Initiating oxygen administration

1, 3 & 4 Therapeutic management is based on an accurate description of the seizure. Turning the child on one side or the other allows drainage of secretions that cannot be swallowed during the seizure. The first safety precaution is to prevent injury by raising the padded side rails. It is impossible to take vital signs during a seizure. Administering oxygen is useless because the child does not breathe during a seizure.

A Spanish-speaking client is being cared for by English-speaking nursing staff. Which communication technique would be correct for the nurse to use when discussing health care decisions with the client? 1. Contact an interpreter provided by the hospital 2. Contact the clients family member to translate for the client 3. Communication with the client using Spanish phrases the nurse learned in a college coarse 4. Communication with the client with the use of a hospital approved Spanish dictionary

1. Contact an interpreter provided by the hospital * Interpreters provided by the health care organization should be used to communicate with clients with limited English proficiency to ensure accuracy of communicated information. In hospital settings, it is not suitable for family members to translate health care information, but they can assist with ongoing interactions during the clients care. The other options do not ensure accurate interpretation of language.

The nurse on the medical-surgical unit tells others staff members, "That client can just wait for the lorazepam; I get so annoyed when people drink to much." Which does this nurse's comment reflect? 1. Demonstration of a personal bias 2. Problem-solving based on assessment 3. Determination of client acuity to set priorities 4. Consideration of the complexity of client care

1. Demonstration of a personal bias * When nurses make judgmental remarks and clients needs are not placed first, the standards of care are violated and quality of care is compromised. Assessments would be objective, not subjective and biased. There is no information about the clients acuity to come to this conclusion regarding priorities. The statement does not reflect information about complexity of care.

Which describes the purpose of the Nurse Practice Acts? 1. Describe and define the legal boundaries of nursing practice within each state 2. Reflect the knowledge and skills possessed by nurses practicing in their profession 3. Legal requirements that describe the minimum acceptable nursing care 4. Protect individuals from losing their health insurance when changing jobs by providing portability

1. Describe and define the legal boundaries of nursing practice within each state * The Nurse Practice Act describe and define the legal boundaries of nursing practice within each state. They help distinguish between nursing and medical practice and establish education and licensure requirements for nurses. Standards of care reflect the knowledge and skills possessed by nurses who are active practitioners in their profession. Standards of care are legal requirements that define the minimum acceptable nursing care. The Health Insurance Portability and Accountability Act (HIPAA) protects individuals from losing their health insurance when changing jobs by providing portability.

Which is the role of the nurse explaining the reason for the intravenous infusion and kit to the client? 1. Educator 2. Manager 3. Advocate 4. Caregiver

1. Educator * The nurse assumes the role of educator when explaining to the client that need for an intravenous infusion. The nurse as a manager oversees the budget of a specific nursing unit or agency and is also responsible for coordinating the activates of the staff providing nursing care. As an advocate, the nurse protects the human and legal rights of the client. The nurse empowers the client with information required to make important health care decisions. The nurse is a caregiver when helping the client maintain and regain health, manage disease symptoms, and achieve a maximum level of functioning.

When caring for a client who is receiving enteral feedings, the nurse would take which measure to prevent aspiration? 1. Elevate the head of the bed between 30 and 45 degrees 2. Decrease flow rate at night 3. Check for residual daily 4. Irrigate regularly with warm tap water

1. Elevate the head of the bed between 30 and 45 degrees * To prevent aspiration, the nurse would keep the head of the bed elevated between 30 and 45 degrees. Elevating the head an higher causes increased sacral pressure and increases the risk of skin breakdown. Decreasing flow rate, checking for residual, and irrigating regularly will not prevent aspiration.

The nurse is discussing discharge plans with a client. The client stated, "I'm worried about going home." The nurse responds, "Tell me more about this." Which interviewing technique did the nurse use? 1. Exploring 2. Reflecting 3. Refocusing 4. Acknowledging

1. Exploring * Exploring is a technique used to obtain more information to better understand the nature of the clients statement. Reflecting is a technique used to either reiterate the content or feeling message. In content flection (paraphrasing), the nurse repeats basically the same statement; in feeling reflection, the nurse verbalized what seems to be implied about feelings in the comment. Refocusing is bringing the client back to a previous point there is no information that this was discussed previously. Acknowledging is providing recognition for a change in behavior, an effort a client has made or a contribution to a discussion.

Which is the goal of school health nursing programs? 1. Health promotion 2. Disease management 3. Chronic care management 4. Environmental surveillance

1. Health promotion *The goal of school health nursing programs is health promotion through the school curriculum. A class on nutritional planning for parents contributes to health promotion. Disease management is one of the many programs of community health centers. These centers provide primary care to a specific client population within a community. Nurse-managed clinics provide nursing care management. The occupational health nurse may conduct environment surveillance for health promotion and accident prevention in the work setting.

A client who is in the advanced stages of illness asks the nurse to contact pastoral services for support. According to the Macmillan model, which is the correct nursing intervention? 1. Immediately involve pastoral services while caring for the client 2. Involve the family member in the client's care instead of pastoral support 3. Listen to the client's request for support and then carry on with the clinical work 4. Falsely promise that pastoral services have been contacted and plan to see the client

1. Immediately involve pastoral services while caring for the client *The Macmillan nurse usually has the knowledge of advanced practice and possesses specialty training. This practice enhances the nurse to gain an in-depth knowledge about spiritual, social, and psychological needs and the pathophysiology of clients living with advanced diseases. The nurse involves pastoral services while caring for the client. Involving a family member may decrease anxiety in the client but may not fulfill the wishes of the client. Just listening to the client's request without implementation of giving false promises can cause loss of trust by the client.

The parents of a 14-month-old boy with bilateral cryptorchidism ask the nurse in the pediatric clinic why it is important for him to have surgery before he is 2 years old. Before responding, the nurse takes into consideration the fact that uncorrected cryptorchidism can result in: 1.Infertility 2.Hydrocele 3.Varicocele 4.Epididymitis

1. Infertility Undescended testes (cryptorchidism) is the failure of the testes to move down the inguinal canal into the scrotum; this migration begins around the 25th to 30th week of gestation. Undescended testes are exposed to body heat that can destroy the sperm-producing ability of the testes, resulting in sterility. A hydrocele is an enlargement of the scrotum with fluid; it is not related to cryptorchidism. A varicocele is a dilation and tortuosity of the scrotal veins; it is not caused by undescended testicles. Inflammation of the epididymis may occur whether or not cryptorchidism is corrected.

An infant who has been found to have developmental dysplasia of the hip (DDH) is being examined in the pediatric clinic. What clinical finding does the nurse expect to identify during the physical assessment? 1.Limited abduction of the affected hip 2.Downward and inward rotation of the affected hip 3.Inability to flex and extend the hip on the affected side 4.Free abduction of the affected hip when placed in the frog position

1. Limited abduction of the affected hip Abduction of the hip is limited because the head of the femur slips out of the acetabulum and is unable to rotate. Rotation of the affected hip is unaffected in an infant with DDH. The hip can be flexed on the affected side. Free abduction of the affected hip is impossible; the frog position may be used in the treatment of DDH.

A client does not take his or her medication regularly and is depressed. Which inference can the nurse make about the client's motivational level? 1. Not motivated 2. Intrinsically motivated 3. Extrinsically motivated with self determination 4. Extrinsically motivated without self-determination

1. Not motivated *If the client is not motivated, then the client may not attempt to eradicate the illness and feel depressed because of the illness. If the client is intrinsically motivated, then the client shows more interest in taking the medications on his or her own rather then because of pressure from other individuals. The client is motivated extrinsically with or without self-determination when he or she may take medication regularly when reminded to do so or when pressured by others.

Which member of the inter-professional team in a palliative care setting serves as the client advocate, evaluating the physical, emotional, and spiritual needs of the client? 1. Nurse 2. Pharmacist 3. Music therapist 4. Primary health care provider

1. Nurse *In a palliative care setting, the health care team would comprise professionals of various disciplines to help achieve care outcomes. The nurse on the inter-professional team evaluates the physical, emotional, and spiritual needs of the client and provides referrals to other members of the team. The primary health care provider assesses the client manifestations of the client. The pharmacist supports the care of the client and the needs of the family. Music therapists help increase the comfort of the client.

Which nursing theory is the inspiration for self-care? 1. Orem's theory 2. Leininger's theory 3. Henderson's theory 4. Neuman's theory

1. Orem's theory * Orem's self-care deficit theory focuses on the client's self-care needs. Orem defines self-care as a learned, goal oriented activity directed toward the self in the interest of maintaining life, health, development, and well-being. Leininger's theory is based on providing care consistent with nursing's emerging science and knowledge, with caring as the central focus. Henderson's theory involves working interdependently with other health care workers. Neuman's theory is based on stress and the client's reaction to the stressor.

How are profits use din a for-profit care organization? 1. Profits are paid out to shareholders 2. Profits are used to buy new equipment 3. Profits are used to build additional facilities 4. Profits are invested in improving health care services

1. Profits are paid out to shareholders * Health care organizations can be classified as for-profit and not-for-profit based on how the profits are distributed. In a for-profit organization, the profits are generated for the shareholder. In a not-for-profit organization, the profits are used to buy new equipment, build additional facilities, and improved health care services.

Which nursing practice is associated with a self-regulation skill? 1. Reflecting on one's experience 2. Reflecting on one's own behavior 3. Supporting one's findings and conclusions 4. Clarifying any data that one is uncertain about

1. Reflecting on one's experience * Self regulation involves reflecting on the nurse's experience. Evaluation involves reflecting on the nurse's own behavior. Explanation involves supporting findings and conclusions. Interpretation involves clarifying any data about which the nurse is uncertain.

Which signs and symptoms are observed in the human body with a decrease in body temperature? Select all that apply. One, some, or all responses may be correct. 1. Shivering 2. Profuse sweating 3. Flushed appearance 4. Dilation of blood vessels 5. Contraction of blood vessels

1. Shivering 5.Contraction of blood vessels *A client who has decreased body temperature may experience shivering due to contraction of the blood vessels in the body. The client who has decreased body temperature may not experience profuse sweating, flushed appearance, and dilated blood vessels. These signs and symptoms appear with an increase in body temperature.

A nurse is assessing a 3-week-old infant who has been admitted to the pediatric unit with hydrocephalus. What finding denotes a complication requiring immediate attention? 1.Tense anterior fontanel 2.Uncoordinated eye/muscle movement 3.Larger head circumference than chest circumference 4.Inability to support the head while in the prone position

1. Tense anterior fontanel A tense or bulging fontanel is indicative of increased intracranial pressure, which is caused by the fluid accumulation associated with hydrocephalus. Conjugate gaze does not occur until 3 to 4 months of age, once the eye muscles have matured. The head is the largest part of the body at this age; the head circumference should be about 1 inch larger than that of the chest. An infant cannot support the head before 1 to 1½ months of age.

Under which type of health care services would the nursing student include subacute care? 1. Tertiary care 2. Continuing care 3. Restorative care 4. Secondary acute care

1. Tertiary care * The nursing student would include subacute care under tertiary care. Subacute care is not a part of continuing care, restorative care, or secondary acute care health care services.

A nurse is helping a 7-year-old child with juvenile idiopathic arthritis (JIA) perform range-of-motion exercises. What outcome indicates that the exercises have been effective? 1.The knees are more mobile. 2.The pedal pulses become stronger. 3.Subcutaneous nodules at the joints recede. 4.The child states that the pain is diminished.

1. The knees are more mobile. The exercises are done to preserve function by mobilizing restricted joints. Circulation is not affected by the arthritic process. Exercises are done to restore joint function; they do not necessarily relieve pain. Exercise does not affect the subcutaneous nodules in the joints.

A client has been placed used insulin needles in a container sealed with heavy-duty tape. Where would the nurse tell the client to dispose of the container? 1. The local hazardous waste collection site 2. The regular household trash 3. The local health department for disposal 4. The Environmental Protection Agency (EPA) through the mail

1. The local hazardous waste collection site * Each stated (province) has its own waste management guidelines for proper disposal of sharps containers, as well as hazardous waste collection sites. Clients cannot place needles in the regular household trash because sharps were considered medical waste. The local health department does not collect sharps containers. Sharps containers are not mailed directly to the EPA.

A client is likely to undergo reconstructive surgery for which purpose? 1. To restore function and/or appearance 2. To replace an organ or tissue 3. To relieve or reduce symptoms 4. To remove or excise an organ or tissue

1. To restore function and/or appearance * The main function of reconstructive surgery is to restore function and/or appearance. This type of surgery includes plastic surgery, a term that is interchangeable with reconstructive surgery. In reconstructive surgery, repairs are made and malformations corrected that are congenital, a result of disease processes, or form traumatic injury. Replacement of a tissue or organ is known as transplant; surgery to relieve or reduce symptoms is known as palliative; and surgery to remove or excise an organ or tissue is known as resection.

Which pulse site is used to perform Allen test? 1. Ulcer 2. Brachial 3. Femoral 4. Dorsalis pedis

1. Ulnar * The ulnar pulse site is used to perform Allen test. The brachial pulse site is used to asses the status of circulation to the lower arm and to auscultate blood pressure. The femoral site is used to assess the character of the pulse during physiological shock or cardiac arrest. The dorsalis pedis site is used to assess the status of circulation in the foot.

When providing postoperative teaching, which rationale would the nurse give to explain the purpose of administering an opioid analgesic via epidural catheter? 1 Facilitates oxygen use 21Relieves abdominal pain 3Decreases anxiety and restlessness 4 Dilates coronary and peripheral blood vessels

2

The registered nurse is teaching the mother of an infant about the prevention of sudden infant death syndrome (SIDS). Which statement by the mother indicates effective learning? 1 "I will sleep in the bed with my baby." 2 "I will position my baby on the back while sleeping." 3 "I will use warm blankets and sheets to cover my baby." 4 "I will put my baby to sleep on soft bedding with pillows."

2 "I will position my baby on the back while sleeping." Sleeping on the stomach may cause upper airway obstruction and increase the risk of SIDS. Therefore, infants should be placed on their backs to decrease this risk. Sleeping in the same bed as the baby may increase the risk of SIDS. Covering the baby with warm blankets may cause overheating or suffocation and can increase the risk of SIDS. The use of soft bedding and pillows for an infant may lead to suffocation. Therefore, the nurse will teach the mother to use a firm mattress.

During the assessment of a newborn, the nurse pulls the infant from a lying to a sitting position and observes that the infant is unable to control the head in an upright position. Which problem does the nurse suspect? 1 Hydrocephalus 2 Down syndrome 3 Congenital syphilis 4 Thromboembolic condition

2 Down syndrome An infant's inability to control the head in an upright position when pulled to a sitting position indicates head lag. The nurse suspects Down syndrome in this neonate because infants with Down syndrome typically experience delays in certain areas of development, including head lag. Hydrocephalus is suspected if the infant has a head circumference more than 4 cm larger than the chest circumference, or if physiologic craniotabes is found in the infant. The presence of physiologic craniotabes in an infant may indicate congenital syphilis. An underlying thromboembolic condition may be suspected if the infant has hypertension.

Which intervention, if followed by the parent, may help manage encopresis in a 9-year-old child? 1 Limiting fluids in the child's diet 2 Including cereals in the child's diet 3 Giving milk and milk-based products 4 Encouraging the child to delay defecation when the urge is felt

2 Including cereals in the child's diet A child with encopresis may pass feces voluntarily or involuntarily at inappropriate settings. Encopresis is usually associated with constipation. Therefore, cereals should be included in the diet to prevent constipation. Limiting fluids may increase the risk of dehydration and constipation. Increasing the fluid intake may help prevent constipation and encopresis. Milk and milk-based products may increase the risk of encopresis and should be avoided. Delaying defecation may result in constipation, and should not be encouraged.

While caring for a newborn in a neonatal intensive care unit, the nurse notices that the neonate is not moving his legs simultaneously. Which condition might the nurse suspect to be the reason for this? 1 Rickets 2 Spinal cord injury 3 Caput succedaneum 4 Neonatal abstinence syndrome

2 Spinal cord injury The spinal cord controls the movements of the legs. Therefore, failure to move the legs indicates a spinal cord lesion or injury. Rickets is suspected if physiologic craniotabes is present in an infant. Caput succedaneum refers to swelling of the infant's head that is sustained during its passage through the birth canal. Neonatal abstinence syndrome is suspected if the infant has sustained rhythmic tremors, twitches, and myoclonic jerks.

Which is observed in the preoperational stage of Piaget's cognitive development? 1 Inductive reasoning 2 Transductive reasoning 3 Sense of cause and effect 4 Deductive and abstract reasoning

2 Transductive reasoning Transductive reasoning is observed in the preoperational stage, in which the individual thinks that because two events occur together, they cause each other. Inductive reasoning occurs in the stage of concrete operations. Sense of cause and effect is observed in the sensorimotor stage of cognitive development. Deductive and abstract reasoning is the characteristic of formal operations.

The registered nurse is teaching a nursing student about caring for a client who has difficulty speaking English. Which statement made by the nursing student would cause communication problems with the client? 1 "I will give the client a call bell." 2 "I will involve the client's family members as interpreters." 3 "I will provide a dictionary to the client if the client can read." 4 "I will use boards and pictures to communication with the client."

2 "I will involve the client's family members as interpreters."

How would the nurse incorporate the quality of accuracy into client documentation? 1 By providing a logical order for the communication 2 By using exact measurements for each client's activity 3 By providing complete and appropriate information in each client's record Incorrect 4 By recording descriptive and objective information that he or she sees, hears, feels, and smells

2 By using exact measurements for each client's activity

What clinical indicators should a nurse expect when assessing a client with hyperthyroidism? Select all that apply. 1. Dry Skin 2. Weight loss 3.Tachycardia 4.Restlessness 5.Constipation 6.Exophthalmos

2, 3, 4 & 6 Weight loss is associated with hyperthyroidism because of the increase in the metabolic rate. Muscle weakness and wasting also occur. Tachycardia, palpitations, increased systolic blood pressure, and dysrhythmias occur with hyperthyroidism because of the increased metabolic rate. Restlessness and insomnia are also associated with hyperthyroidism because of the increased metabolic rate. Protrusion of the eyeballs occurs with hyperthyroidism because of peribulbar edema. Dry, coarse, scaly skin occurs with hypothyroidism, not hyperthyroidism, because of decreased glandular function. Smooth, warm, moist skin occurs with hyperthyroidism. Constipation is associated with hypothyroidism. Increased stools and diarrhea are associated with hyperthyroidism.

A registered nurse is explaining the importance of capitation to a nursing student. What information should the nurse provide? Select all that apply. 1 Capitation is used to review the quality, quantity, and cost of hospital care. 2 Capitation influences the way healthcare providers deliver care in all types of settings. 3 Capitation means that primary healthcare providers are paid a fixed amount per client of a health care plan. 4 Capitation identifies and eliminates the overuse of diagnostic and treatment services ordered by primary healthcare providers for Medicare. 5 Capitation aims to build a payment plan for select diagnoses or surgical procedures that consists of the best standards of care at the lowest cost.

2,3,5

The nurse records the clients weight and body mass index (BMI) at a health range, but the client states, "I wish I were as thin as my coworkers." Which culturally bound condition is the client at risk for? 1. Neurasthenia 2. Anorexia nervosa 3. Shenjing shuairuo 4. Ataque de nervios

2. Anorexia nervosa * Anorexia nervosa is a Western culture-bound eating disorder characterized by obsession with body image. A client who continues to follow weight loss diets despite being a healthy weight may be at risk for malnutrition. The client with neurasthenia may feel a lack of energy but not necessarily from following a strict diet to maintain body image. Shenjing shuairuo is a condition associated with Chinese culture that focuses on a weakness of nerves and is not associated with eating disorders or body image. Ataque de nervios is a Latin0-Caribbean culture-bound syndrome and is not associated with body image.

Which is the action of an antidiuretic hormone (ADH)? 1. Reduces blood volume 2. Decreases water lose in urine 3. Increases urine output 4. Initiates the thirst mechanism

2. Decreases water loss in urine * ADH is released by the posterior pituitary gland. It is released mainly in response to wither a decrease in blood volume or an increased concentration of sodium or other substances in the plasma. It acts to decrease the production of urine by increasing the reabsorption of water by renal tubules. A decrease in ADH would cause reduced blood fluid volume; decreased ability of the kidneys to reabsorb water, resulting in increased urine output; and an increase in the thirst mechanism.

Which factor is used to assess the quality of health care provided to a client? 1. Fall-prevention measures employed for the client 2. Functional health status of the client after discharge 3. Hand hygiene practiced by the health care personnel 4. Teamwork and coordination among health care personnel

2. Functional health status of the client after discharge * Health care providers determine the quality of care provided to the client by measuring outcomes that show how the clients health status has changed. one method of measuring the quality of health care provided to the client after discharge. The nursing staff would take necessary fall prevention measures for the client; however, this is not a measurable outcome. All health care personnel would practice hand hygiene to prevent infection, which is a quality measure, not an outcome of health care. Teamwork and coordination among health care personnel are important to provide efficient health care to the client. They are not outcomes of health care.

The nurse changing the dressin on the clients perineum would fall into which zone? 1. Public zone 2. Intimate zone 3. Personal zone 4. Vulnerable zone

2. Intimate zone * Changing a clients dressing on the perineum, falls under the intimate zone. For this action, the appropriate interpersonal distance between the nurse and the client would be between 0 and 19 inches. The nurse lecturing a class of students or speaking at a community forum lies within a public zone. A personal zone refers to the nurse sitting on the clients bedside, taking a clients history, or teaching a client individually. The vulnerable zone is where special care is needed.

Which statement accurately describes a health care policy as it relates to health care economics? 1. It related to maintaining standards of health care and achieving outcomes related to the goals of health care. 2. It provides overarching goals and helps in setting priorities and values for the distribution of health resources. 3. It governs the insurance industry and plays a very important role in the application of health care funding and reform. 4. In involves the collaboration of health care workers and other resources required to perform all required client care activities.

2. It provides overarching goals and helps in setting priorities and values for the distribution of health resources. * A health care policy provides overarching goals and helps in setting priorities and values for the distribution of health resources. Health care quality related to issues surrounding standards of care and health care outcomes. Health care coordination involves the collaboration between health care professionals and other resources, which are required to deliver client care. Health care law is a governing factor in health care economics and plays a vital role in the application of health care funding and reform.

Which database can be used to find studies related to allied health sciences? 1. EMBASE 2. MEDLINE 3. National Guidelines Clearinghouse 4. Cochrane Database of Systemic Reviews

2. MEDLINE *The MEDLINE database includes studies in medicine, nursing dentistry, psychiatry, veterinary medicine, and allied health. EMBASE includes biochemical and pharmaceutical studies. The National Guideline Clearinghouse includes a repository for structured abstracts about clinical guidelines and their development. It also includes a condensed version of the guidelines. The Cochrane Database includes full text of regularly updated systemic reviews prepared by the Cochrane Collaboration as well as completed reviews and protocols.

A healthcare team is caring for a client who is hospitalized with severe diarrhea. According to the principle of right of supervision of delegation, what does the RN do in this situation? 1. Feed the client every 2 hours. 2. Monitor the hemodynamic status. 3. Assess the resources required in the client's care. 4. Follow the institutional procedures for administering fluids.

2. Monitor the hemodynamic status.

A nurse plans to discuss childhood nutrition with a group of parents whose children have Down syndrome in an attempt to minimize a common nutritional problem. What problem should be addressed? 1.Rickets 2.Obesity 3.Anemia 4.Rumination

2. Obesity Obesity is a common nutritional problem of children with Down syndrome. It is thought to be related to excessive caloric intake and impaired growth. Rickets is a nutritional disorder related to vitamin D deficiency; it is usually not encountered in these children. Anemia is the most common nutritional problem in children with iron deficiency. Rumination is an eating disorder of infancy characterized by repeated regurgitation without a gastrointestinal illness.

The parents of an 18-month-old toddler are anxious to know why their child has experienced several episodes of acute otitis media. What should the nurse explain to the parents about why toddlers are prone to middle ear infections? 1.Immunological differences between adults and young children 2.Structural differences between eustachian tubes of younger and older children 3.Functional differences between eustachian tubes of younger and older children 4.Circumference differences between middle ear cavity size of adults and young children

2. Structural differences b/w Eustachian tubes of younger and older children. The eustachian tube in young children is shorter and wider, allowing a reflux of nasopharyngeal secretions. Immunological differences are not a factor in the development of otitis media. There is no difference in the function of the eustachian tube among age groups. The size of the middle ear does not play a role in the occurrence of otitis media in young children.

Which statement defines the term "family resiliency?" 1. Each family is unique 2. The family has an ability to cope with stressors 3. An interfamilial structure and support system exist 4. The family has the ability to transcend lifestyle changes

2. The family has an ability to cope with stressors * Family resiliency is the ability of the family to cope with expected and unexpected stressors. Family diversity is the uniqueness of each family. Family durability is the interfamilial support system that extents beyond the walls of the household. The parents of this family may remarry or children may leave the home as adults; however, the family is capable of transcending inevitable lifestyle changes.

Which nursing action is a part of the evaluation phase of the critical thinking process? 1Collecting all the data in order 2Looking at all the situations objectively 3Support the findings and drawing conclusions 4Be open-minded to information about a client 5Using several criteria to determine the effectiveness of a nursing intervention

2Looking at all the situations objectively 5Using several criteria to determine the effectiveness of a nursing intervention

How many levels of critical thinking exist for nursing judgment?

3

On her first visit to the neonatal intensive care unit to see her preterm newborn, the mother's only comment to the nurse is, "My baby looks so fragile. Do you think my child will make it?" Which is the most appropriate response by the nurse? 1 "Many infants born as small as yours have done just fine." 2 "The staff is confident in your child's prognosis because preterm babies do look like this at first." 3 "Understandably, your baby looks fragile to you. What have you learned about the condition?" 4 "Your baby is not as fragile as it appears. Do you find it so frightening that you cannot touch your child?"

3 "It's understandable that your baby looks fragile to you. What have you learned about the condition?"

During a routine checkup, the nurse learns that an adolescent patient is planning to get a navel piercing. What should the nurse assess in the adolescent to ensure safety? 1 Thyroxin levels 2 Hemoglobin levels 3 Blood glucose levels 4 Serum potassium levels

3 Blood glucose levels Skin piercing can cause bleeding, dermatitis, and metal allergy. A patient with diabetes mellitus has an increased risk of skin infection due to high blood glucose levels. Therefore, the nurse should monitor the adolescent's blood glucose levels to ensure safety. A change in thyroxin levels does not indicate that the client has a risk of bleeding, so the nurse does not assess thyroid levels. The nurse assesses hemoglobin levels when the patient has risk of anemia. The nurse will assess serum potassium levels if the adolescent has risk of dehydration, but not before skin piercing.

What common finding can the nurse identify in most children with symptomatic cardiac malformations? 1 Mental retardation 2 Inherited genetic factors 3 Delayed physical growth 4 Clubbing of the fingertips

3 Delayed physical growth Children with cardiac malformations often require more energy to fulfill the activities of daily living; decreased oxygen utilization and increased energy output in the developing child result in a slow growth rate. Mental retardation is not a common finding in children with congenital heart disease. Cardiac anomalies are more often a result of prenatal, rather than genetic, factors. Clubbing is not characteristic of most children with cardiac anomalies, only of those with more severe hypoxia.

Upon interacting with the parent of an infant, the nurse observes that the parent is using an incorrect formula preparation method. Which risk does this pose to the infant? 1 Colic 2 Plagiocephaly 3 Failure to thrive 4 Sudden infant death syndrome

3 Failure to thrive Incorrect formula preparation can lead to inadequate calorie intake and malnutrition, which causes failure to thrive in the infants. Colic in infants may be due to overfeeding, improper feeding techniques, and swallowing excessive air, but not incorrect formula preparation. Positional plagiocephaly and sudden infant death syndrome (SIDS) are avoided by repositioning the infant's sleeping positions, not through formula preparations.

The school health nurse is teaching a group of teachers about promoting the mental health of school-age children. Which action made by the teachers promotes a sense of industry among the children? 1 Separating children during tasks 2 Basing a reward structure on evidence of mastery 3 Giving grades and gifts for satisfactory performances 4 Comparing the performances of children with one another

3 Giving grades and gifts for satisfactory performances During the psychosocial development of school-age children, reinforcement in the form of grades, material rewards, additional privileges, and recognition provides encouragement and stimulation. A sense of accomplishment also involves the ability to cooperate, to compete with others, and to cope effectively with people, so separating children will not promote their mental health. When the reward structure is based on evidence of mastery, children who are incapable of developing these skills are also at risk for feelings of inadequacy and inferiority. Comparison with one another can also cause some children to develop negative feelings towards themselves, and result in a sense of inferiority.

A 6-week-old infant and his mother arrive in the emergency department in an ambulance. The father arrives several minutes later with two children, 7 and 9 years old. The infant is not breathing, and the eventual diagnosis is sudden infant death syndrome (SIDS). The parents take turns holding the infant in another room. The nurse remains present and provides emotional support to the parents. What is an important short-term goal for this family? 1 Identifying the problems that they will be facing as a result of the loss of the infant 2 Accepting that there was nothing that they could have done to prevent the infant's death. 3 Including the infant's siblings in the events and grieving in the wake of the infant's death 4 Seeking out other families who have lost infants to SIDS and obtaining support from them.

3 Including the infant's siblings in the events and grieving in the wake of the infant's death The other children need to be involved with the grieving process and to work through their own feelings. Identifying the problems that the family will be facing in regard to the loss of the infant is a long-term goal. It is too early to seek out other families who have lost infants to SIDS and receive support from them. It is premature to accept that there was nothing that the family could have done to prevent the infant's death; in fact, they may never achieve this goal.

Which attribute of temperament is related to the energy level of the child's reaction? 1 Attention span 2 Sensory threshold 3 Intensity of reaction 4 Approach-withdrawal

3 Intensity of reaction The energy level of the child's reaction is called intensity of reaction. Sensory threshold is the amount of stimulation, such as sounds or light, required to evoke a response in the child. Approach-withdrawal is the nature of initial responses to a new stimulus. Approach responses are positive expressions, and withdrawal responses are negative expressions. The length of time a child pursues a given activity is called attention span.

What does a nurse recognize as the most serious complication of meningitis in young children? 1 Epilepsy 2 Blindness 3 Peripheral circulatory collapse 4 Communicating hydrocephalus

3 Peripheral circulatory collapse Peripheral circulatory collapse (Waterhouse-Friderichsen syndrome) is a serious complication of meningococcal meningitis caused by bilateral adrenal hemorrhage. The resultant acute adrenocortical insufficiency causes profound shock, petechiae, ecchymotic lesions, vomiting, prostration, and hypotension. Although epilepsy or blindness may occur, neither condition is as serious a complication as peripheral circulatory collapse. Similarly, although hydrocephalus may occur, it is rare and not as serious as peripheral circulatory collapse.

After teaching from the nurse about common infant injuries, the parent says, "I will not allow my child to go near the plants in our house." Which risk to the infant can be prevented by this action? 1 Falls 2 Asphyxia 3 Poisonings 4 Allergic reactions

3 Poisonings Household plants may be a source of accidental poisonings because a curious infant may put the leaves in the mouth. Therefore, plants should be kept out of the child's reach to avoid poisonings and ingestions. Stairs, diaper changing table, and infant walkers are the risk factors for falls in the infant. Asphyxia can be prevented by keeping the small objects out of reach of an infant and avoid giving hard candies to the infants and toddlers. Allergic reactions can be prevented by preventing exposure to the allergens.

The parent of a child says, "My child is repeatedly banging the table to make loud sounds." Which sensorimotor stage of cognitive development best explains this behavior of the child? 1 Reflexes 2 Primary circular reactions 3 Secondary circular reactions 4 Coordination of secondary schemas

3 Secondary circular reactions The third stage of Piaget's sensorimotor phase involves secondary circular reactions, in which the child intentionally repeats an action in order to trigger a response. The first stage of sensorimotor phase comprises of reflexes, in which the infant will exhibit involuntary responses to stimuli, such as sucking, rooting, grasping, and crying. Replacement of reflexive behavior with voluntary acts is seen in primary circular reactions, which is the second stage of sensorimotor phase. The fourth stage is coordination of secondary schemas, in which the child starts showing intentional actions and uses previous behavioral achievements, primarily as the foundation for new intellectual skills.

Which internal asset helps young people make positive choices and build relationships? 1 Positive values 2 Positive identity 3 Social competencies 4 Commitment to learning

3 Social competencies Social competencies help young people make positive choices and build relationships. Positive values are a strong sense of values that are needed to direct the choices of young people. Positive identity provides a sense of own power, purpose, worth, and promise in young people. Young people need to develop a commitment to education and lifelong learning.

The nurse is reinforcing best parenting practices to the parents of a 13-year-old child. Which statements made by the parent need correction? Select all that apply. 1 "I should apologize to my child when I am wrong." 2 "I should assist my child in selecting appropriate career goals." 3 "I should motivate my child to perform well in exams by comparing her to her siblings." 4 "I should strictly instruct my child to adhere to the house rules even though it hurts her." 5 "I should teach my child to make decisions and understand consequences as an adult would."

3 "I should motivate my child to perform well in exams by comparing her to her siblings." 5 "I should teach my child to make decisions and understand consequences as an adult would." The nurse should suggest that the parents avoid comparing the adolescent to his or her siblings in order to prevent the development of an inferiority complex. The nurse should encourage parents to allow adolescent children to make their own choices and learn from them, even when those choices are not the choices an adult would make. The nurse should suggest that the parents respect their child and apologize to their child if they make a mistake. The nurse should suggest that the parents assist their child in selecting appropriate career goals and preparing for adult roles. The nurse should suggest that the parents make clear house rules and instruct the child to adhere to them, as it helps prevent the development of high-risk behaviors, like alcohol addiction.

Which of these measures does a nurse take during the working phase of a helping relationship? 1 Evaluating goal achievements with the client 2 Anticipating health concerns or issues that may arise 3 Using appropriate self-disclosure and confrontation 4 Prioritizing the client's problems and identifying his or her goals

3 Using appropriate self-disclosure and confrontation

A client is placed on a stretcher and restrained with straps while being transported to the x-ray department. A strap breaks, and the client falls to the floor, sustaining a fractured arm. Later the client shows the strap to the nurse manager, stating, "See, the strap is worn just at the spot where it snapped." What is the nurse's accountability regarding this incident? 1 Exempt from any lawsuit because of the doctrine of respondeat superior 2 Totally responsible for the obvious negligence because of failure to report defective equipment 3 Liable, along with the employer, for misapplication of equipment or use of defective equipment that harms the client 4 Exonerated, because only the hospital, as principal employer, is responsible for the quality and maintenance of equipment

3 liable

Which age is considered the phallic stage according to Sigmund Freud's developmental theory?

3 to 6 years old

The nurse speaking in support of the best interest of a vulnerable client reflects which nursing duty? 1. Caring 2. Veracity 3. Advocacy 4. Confidentiality

3. Advocacy * The nurse has a professional duty to advocate for a client by promoting what is best for the client. This is accomplished by ensuring that the client's needs are met and by protecting the client's rights. Caring is a behavioral characteristics of the nurse. Veracity relates to the habitual observance of truth, fact, and accuracy. Confidentiality is an ethical principle and legal right that the nurse will hold secret all information relating to the client unless the client gives consent to permit disclosure.

Which describes the role of the nurse in this situation when he or she informs the health care provider the client is requesting pain medication after surgery? 1. Educator 2. Manager 3. Advocate 4. Administrator

3. Advocate *The nurse acts as a client advocate by speaking to the primary health care provider on behalf of the client. The nurse acts as an educator while teaching the client facts about health and the need for routine care activities. The nurse manager used appropriate leadership styles to create a nursing environment for client-centered care. The nurse administrator manages client care and delivery of specific nursing services with a health care agency.

Which organization assists in establishing policies related to Medicare and Medicaid payment for meaningful use of electronic health records (EHRs)? 1. National Institutes of Health (NIH) 2. American Medical Informatics (AMIA) 3. Center for Medicare and Medicaid Services (CMS) 4. Health Information Management Systems Society (HIMSS)

3. Center for Medicare and Medicaid Services (CMS) * CMS rules specify how health care facilities and providers make meaningful use of the EHRs and technologies to receive payment from Medicare and Medicaid. The NIH uses translational bioinformatics for medical research. The AMIA and the HIMSS have been involved in identifying nursing informatics competencies

Which of these programs is least likely to focus on medication delivery process modification? 1. Evaluation research 2. Quality improvement 3. Experimental research 4. Performance improvement

3. Experimental research *Experimental research is least likely to focus on medication delivery process modification. Quality improvement, evaluation research, and performance improvement are all likely to focus on medication delivery process modification to make the process better for the client.

Which critical thinking skill in nursing practice requires the nurse to possess knowledge and experience for choosing care strategies for clients? 1. Analysis 2. Inference 3. Explanation 4. Interpretation

3. Explanation * Explanation requires knowledge and experience for choosing strategies for care for clients. Analysis is a critical thinking skill that requires open-mindedness while looking at the client's information. the skill of interference is associated with noticing relationship in the findings. Interpretation is associated with order data collection.

The mother of a 2-year-old child tells the nurse that she is concerned about her child's vision. What behavior when the child is tired leads the nurse to suspect strabismus? 1 One eyelid droops. 2Both eyes look cloudy. 3One eye moves inward. 4Both eyes blink excessively

3. One eye moves inward An inward moving eye (tropia) is one form of strabismus. A drooping eyelid is called ptosis; it may be congenital or caused by trauma. Cloudy eyes are associated with congenital cataracts. Blinking may be a tic.

An anxious parent of a 6-year-old child expresses that the child looks slimmer than a year ago, although the nurse finds the child's height and weight to be age-appropriate. What is the nurse's response to the parent? 1 "The child's condition may need further investigation." 2 "It seems like the child has not been eating well lately." 3 "You need to include more carbohydrates in the child's diet." 4 "The fat has diminished, and the fat distribution pattern has changed."

4 "The fat has diminished, and the fat distribution pattern has changed." As the preschooler develops into a school-age child, the fat in the body diminishes and the distribution pattern of the fat changes, making the child appear slim. Looking slimmer during school age is a normal part of growth and development. It does not need further investigation and does not indicate that the child has not been eating well. The child may become obese if the parent includes more carbohydrates in the child's diet.

A father expresses concern that his 2-year-old daughter has become a "finicky eater" and is eating less. How should the nurse respond? 1 "Your daughter has become manipulative." 2 "She's probably experiencing the stress of a typical 2-year-old." 3 "She may have an eating problem that requires a referral to a specialist." 4 "Your daughter's behavior is expected in response to her slower growth."

4 "Your daughter's behavior is expected in response to her slower growth." Growth slows during the toddler years and these children generally do not eat as much as they do during infancy; this is called physiologic anorexia, which is typical of this age group. Toddlers may try to manipulate as they assert their autonomy, but usually not through eating behaviors unless the parents express anxiety and concern over their food intake. Although toddlers have difficulty withstanding frustration and are prone to temper tantrums, these eating behaviors are within the norm for toddlers. Eating disorders usually do not occur in children this young; these behaviors are typical of healthy toddlers.

In which type of play do children play together, share toys, and communicate with each other? 1 Parallel play 2 Solitary play 3 Onlooker play 4 Associative play

4 Associative play Children from 2 years of age are mostly involved in associative play, in which they play together, share toys, and communicate with other children. In parallel play, children play next to each other but have little interaction. In solitary play, the child plays alone, which is usually seen in infants. In onlooker play, a child watches others playing, but does not interact.

An 8-year-old child is being prepared for surgery the next day. How should the nurse present preoperative instructions to this child? 1 By repeating instructions often 2 By providing time for needle play 3 By using several abstract examples 4 By focusing on simple anatomical diagrams

4 By focusing on simple anatomical diagrams According to Piaget, an 8-year-old child's level of development is in the stage of concrete operations; the child will benefit from simple, concrete examples. The preschooler and younger child, not the school-age child, require repetition. Therapeutic needle play is more appropriate if and when the child is to receive an injection. The child who is in the period of concrete operations cannot think in the abstract; the ability to do this develops during adolescence.

While playing, a child takes a few pebbles and places them in order from smallest to largest. Which stage of cognitive development does the child's behavior demonstrate? 1 Sensorimotor 2 Preoperational 3 Formal operations 4 Concrete operations

4 Concrete operations During the stage of concrete operations, the child's thought becomes more logical and coherent. The child in this stage is able to classify, sort, order, and organize facts about the world to use in problem-solving. Therefore, the child placing the pebbles in order of smallest to largest indicates that the child is in the stage of concrete operations. In the sensorimotor stage, the child develops a sense of cause and effect as they direct behavior towards objects. In the preoperational stage, thinking is concrete and tangible. The child lacks the ability to make deductions or generalizations. In formal operations, thought is characterized by adaptability and flexibility. Abstract thinking and problem-solving skills are observed in this stage.

A middle-aged adult contributes to future generations through parenthood, teaching, and community involvement. To which stage of Erikson's theory does this relate? 1 Ego integrity versus despair 2 Intimacy versus isolation 3 Identity versus role confusion 4 Generativity versus stagnation

4 Generativity versus stagnation In the generativity versus stagnation stage, the adult focuses on supporting future generations. Middle-aged adults achieve success in this stage by contributing to future generations through parenthood, teaching, and community involvement. The ego integrity versus despair stage is seen in older adults. During this stage, some older adults live their lives with a sense of satisfaction and others see themselves as failures marked by despair and regret. The intimacy versus isolation stage is seen in young adults, in which they develop a sense of identity and a capacity to love and care for others. In the identity versus role confusion stage, an individual will have a marked preoccupation with his or her appearance and body image.

What behavior does the nurse expect a healthy 5-month-old infant to exhibit? 1 Using the pincer grasp 2 Sitting without support 3 Crawling across the floor 4 Grasping objects voluntarily

4 Grasping objects voluntarily The 5-month-old infant's neurological development has reached the stage at which objects can be grasped voluntarily; this is considered a developmental milestone. The pincer grasp appears between 9 and 12 months of age. Sitting alone without support is usually accomplished at 6 to 8 months of age. The infant begins to crawl at 8 to 10 months of age.

According to Piaget, which developmental stage is characterized by logical thinking? 1 Preschool 2 Adolescence 3 Early childhood 4 Middle childhood

4 Middle childhood Logical thinking is a characteristic of middle childhood, which is 6 to 12 years of age. The use of symbols and egocentric behavior is seen in preschoolers. Adolescence is characterized by abstract thinking. Early childhood is characterized by a preoperational period during which thinking using symbols and egocentric behavior are observed.

While assessing a newborn, the nurse strokes the newborn's cheek and observes for a response. What reflex does the newborn produce in response to the nurse's stimulation? 1 Red reflex 2 Startle reflex 3 Sucking reflex 4 Rooting reflex

4 Rooting reflex When the infant's cheek is stroked, the infant will respond by turning toward the stimulated side, which is called the rooting reflex. The red reflex is elicited by placing the infant in a dark room. In an alert state, many infants open their eyes in a supported sitting position. The sucking reflex is elicited by placing a nipple or gloved finger in the infant's mouth. The startle reflex can be elicited by making a loud noise near the infant.

The nurse finds that an adolescent male's trunk is short when compared to the legs. What should the nurse infer from these findings? 1 The adolescent needs further evaluation. 2 The adolescent needs a lot of physical activity. 3 The adolescent may need hormone pills for growth. 4 The adolescent has normal growth and development.

4 The adolescent has normal growth and development. Adolescents have a characteristic growth pattern. In male adolescents, the extremities grow first, followed by the trunk, which can make them appear awkward, with a short trunk and long limbs. Physical activity will not be beneficial because the long extremities reflect normal growth and development. The adolescent does not need hormone pills or further evaluation because these findings are normal for his age.

The nurse is assessing a clients degree of edema and finds 8mm of depth. Which would be correct of depth. Which would be correct to document? 1. 1+ 2. 2+ 3. 3+ 4.4+

4. 4+ * Edema of 8mm is documented at 4+. If the edema has a depth of 2mm, then it is documented as 1+. If the edema has a depth of 4mm, it is documented at 2+. If the edema has a depth of 6mm, then it is documented as 3+.

A 9-year-old child who has had type 1 diabetes for several years is brought to the emergency department of a community hospital. The child is exhibiting deep, rapid respirations; flushed, dry cheeks; abdominal pain with nausea; and increased thirst. What blood pH and glucose level does the nurse expect the laboratory tests to reveal? 1.7.20 and 60 mg/dL 2.7.50 and 60 mg/dL 3.7.50 and 460 mg/dL 4.7.20 and 460 mg/dL

4. 7.20 and 460 mg/dL A pH of 7.20 and blood glucose level of 460 mg/dL are expected values in ketoacidosis; the pH of 7.20 indicates acidosis (metabolic) and the blood glucose level of 460 mg/dL is higher than the expected range of 90 to 110 mg/dL. Although the blood pH of 7.20 indicates acidosis, the blood glucose of 60 mg/dL is less than the expected range of 90 to 110 mg/dL, indicating hypoglycemia rather than hyperglycemia. Neither the pH of 7.50 nor the blood glucose value of 60 mg/dL is expected with ketoacidosis; with ketoacidosis, the pH is decreased and the blood glucose level is increased. Although the blood glucose is increased with ketoacidosis, the pH is decreased, not increased; a pH of 7.50 indicates alkalosis.

Which theory is based on the model of primacy of caring? 1. Roy's theory 2. Watson's theory 3. Neuman's theory 4. Benner and Wrubel's theory

4. Benner and Wrubel's theory * The model of primacy of caring is the basis of Benner and Wrubel's theory. This theory focuses on clients need for caring as a means of coping with stressors of illness. According to Roy's theory, the goal of nursing is to help the client adapt to changes in physiological needs, self-concept, role function, and interdependent relations during health and illness. Watsons theory of transpersonal caring defines the outcome of nursing activity regarding the humanistic aspects of life. This theory promotes health, restoring the client to health, and preventing illness. Neuman's theory is based on stress and the clients reaction to the stressor.

The nurse applies a cold pack to relieve musculoskeletal pain. Which rational explains the analgesic properties of cold therapy? 1. Promoting analgesic and circulation 2. Numbing the nerves and dilating the blood vessels 3. Promoting circulation and reducing muscle spasms 4. Causing local vasoconstriction, preventing edema and muscle spasm

4. Causing local vasoconstriction, preventing edema and muscle spasm *Cold causes the blood vessels to constrict, which reduces the leakage of fluid into the tissues and prevents swelling and muscle spasms. Cold does promote analgesic but not circulation. It may numb nerves but does not dilate blood vessels. Cold therapy also may numb the nerves and surrounding tissues, this reducing pain.

Which interview technique is the nurse using when asking a client to score his or her pain on a scale from 0 to 10? 1. Probing 2. Back channeling 3. Open-ended questioning 4. Closed-ended questioning

4. Closed-ended questioning *Asking a client to score pain on a scale of 0 to 10 is a type of closed-ended question. These types of questions specify the cause of the problem or the client's experience of the illness. Asking whether anything else is bothering the client is an example of probing. When a client says something, a response by the nurse such as "All right" or "Go on" is called back channeling. This interview technique encourages a client to provide more details. The nurse asks open-ended, nonspecific questions such as "What brought you to the hospital today?" to elicit the client's side of the story. Such questions are related to the client's health history and can strengthen the nurse-client relationship.

A 4-year-old child is admitted to the pediatric unit with a diagnosis of Wilms tumor. Considering the unique needs of a child with this diagnosis, the nurse should place a sign on the child's bed that states: 1.Keep NPO. 2.No IV medications. 3.Record intake and output. 4.Do not palpate the abdomen.

4. Do not palpate the abdomen. Palpation increases the risk of tumor rupture and is contraindicated. There are no data to indicate that surgery is scheduled; therefore there is no reason to maintain nothing-by-mouth (NPO) status. There is no contraindication to intravenous medication. Recording of intake and output may or may not be instituted; it is not specific to children with Wilms tumor.

How will the nurse researcher categorize research in which subjects are given chlorhexidine and povidone-iodine as antiseptics? 1. Evaluation research 2. Descriptive research 3. Correlational research 4. Experimental research

4. Experimental research *The nurse will categorize this study as experimental research. In experimental research, the investigator gives variables randomly to the subjects. In this case subjects are given chlorhexidine and povidone-iodine to test the efficacy in reducing infection. Evaluation research is an initial study that refines a hypothesis, such as testing a new exercise in older clients with dementia. In a descriptive study, the characteristics of a person or a situation are measured. For example, a researcher may examine nurses' bias while caring for obese clients. Correlation research is used to find out the relationship between different variables without the interference of a a researcher. An example is determining the educational status of nurses and their satisfaction with their jobs.

Which legal complication might the nurse face for using a restraint without a legal warrant on a client? 1. Libel 2. Negligence 3. Malpractice 4. False imprisonment

4. False imprisonment * If the nurse uses restraints without a legal warrant on a client, he/she may be charged with false imprisonment. Libel is the written defamation of character. Negligence is any conduct that falls below the standard of care. Malpractice is a type of negligence that is regarded as professional negligence.

Which opposing conflict would a middle aged adult face according to Erikson's theory of psychosocial development? 1. Integrity versus despair 2. Intimacy versus isolation 3. Identify versus role confusion 4. Generativity versus self-absorption and stagnation

4. Generativity versus self-absorption and stagnation * According to Erikson's theory of psychosocial development, a middle-aged adult is likely to face the opposing conflict generativity versus self-absorption and stagnation. An older adult is likely to face the opposing conflict integrity versus despair. A young adult may face the opposing conflict intimacy versus isolation. An adolescent may face the opposing conflict identity versus role confusion.

An abscess develops in an obese adult after abdominal surgery. The wound is healing by secondary intention. Which diet would the nurse expect the health care provider to prescribe to meet this clients immediate nutritional needs? 1. Low in fat and vitamin D 2. High in calories and fiber 3. Low i residue and bland 4. High in protein and vitamin C

4. High in protein and vitamin C * Protein and vitamin C promote wound healing; this is a postoperative priority. Although a low-fat diet is preferred for an obese client, Vitamin D, as well as other vitamins, should not be limited. A high-calorie diet can increase obesity, and there is no indication that this client is at risk for constipation requiring a high-fiber diet. A low-residue bland diet can cause constipation; the priority is for nutrients to promote healing.

Which step in the nursing process would involves promoting a safe environment for the client? 1. Planning 2. Diagnosis 3. Assessment 4. Implementation

4. Implementation * The nurse promotes a safe environment during the implementation stage of the nursing process. During the planning stage, the nurse develops an individualized care plan for the client. The plan contains strategies and alternatives to achieve specific outcomes. During the diagnosis stage, the nurse analyzes the assessment data to determine the health care issues. The nurse collects comprehensive data pertinent to the clients health and situation during the assessment stage.

Which interval variable influences health benefits and practices? 1. Family practices 2. Culture background 3. Socioeconomic factors 4. Intellectual background

4. Intellectual background * Intellectual background is an internal factor that affects the clients health beliefs and practices. A clients knowledge, educational background, and past experiences influence how a client thinks about health. Family practices, cultural background, and socioeconomic factors are among the external factors that influence health beliefs and practices.

The nurse concludes that a client with a body temperature of 98.6 F is experiencing which condition? 1. Hypothermia 2. Hyperpyrexia 3. Hyperthermia 4. Normothermia

4. Normothermia * A body temperature of 98.6 F is normal. The nurse concludes that the client has a normothermia. The client does not have low body temperature or hypothermia. The clients body temperature does not exceed the normal range; therefore, the client does not have hyperpyrexia or hyperthermia.

Which concept refers to respecting the rights of others? 1. Maturity 2. Systematicity 3. Inquisitiveness 4. Open-mindedness

4. Open-mindedness * Open-mindedness refers to respecting the rights of others and being tolerant of different viewpoints. Maturity refers to reflecting on ones own judgements and having cognitive maturity. Systematicity refers to being organized and focused. Inquisitiveness refers to acquiring knowledge.

Which activity places a client at risk for hyperthermia? 1. Snowmobiling 2. Skiing in the winter 3. Hiking Alaskan mountains 4. Performing strenuous activity in high humidity

4. Performing strenuous activity in high humidity *When a client performs strenuous activity in high humidity, it reduces heat loss from the body and results in hyperthermia. Activities such as snowmobiling, skiing, and hiking in cold weather may cause hypothermia because they occur in cold temperatures and may lower the body temperature.

Two nurses are planning to help a client with one-sided weakness move up in bed. Which principle of body mechanics would the nurse observe? 1. Instruct the client to position one arm on each shoulder of the nurses 2. Direct the client to extend the legs and remain still during the procedure 3. Have both nurses shift their weight from the front leg to the back leg as they move the client up in bed 4. Position the nurses on either side of the bed with their feet apart, gather the pull sheet close to the client, turn toward the head of the bed, and then move the client

4. Position the nurses on either side of the bed with their feet apart, gather the pull sheet close to the client, turn toward the head of the bed, and then move the client * Positioning the nurses on either side of the bed with their feet apart, gathering the pull sheet close to the client, turning toward the head of the bed, and then moving the client places both nurses in a stable position in functional alignment, thereby minimizing stress on muscles, joints, ligaments, and tendons. The client should be instructed to fold the arms across the chest; this keeps the client's weight toward the center of the mass being moved and keeps the arms safe during the move up in bed. The nurses would assist the client in flexing the knees and placing the feet flat on the bed; this enables the client to push the body upward using a major muscle group. The client's assistance to the best of his or her ability reduces physical stress on the nurse as they move the client up in bed. On the count of three, weight should be shifted from the back to the front leg, not the front to the back leg. This action generates movement in the direction in which the client is being moved.

A client complains of pain in the ear. While examining the client the nurse finds swelling in front of the left ear. Which lymph node would the nurse expect to be involved? 1. Mastoid 2. Occipital 3. Submental 4. Preauricular

4. Preauricular * The preauricular lymph node is located in front of the ear and in this situation would be edematous. The mastoid or posterior auricular lymph node is present behind the ear. The occipital lymph nodes are located in the back of the head, near the occipital bone of the skull. Submental lymph nodes are located below the chin.

Under which type of health care services would the student nurse include sports medicine? 1. Primary care 2. Tertiary care 3. Preventive care 4. Restorative care

4. Restorative care * The student nurse would include sports medicine under restorative care. It is not categorized as primary, tertiary, or preventive health care services.

A mother brings her 6-year-old child to the pediatric clinic, stating that the child has not been feeling well, is weak and lethargic, and has a poor appetite, headaches, and smoky-colored urine. What additional information should the nurse obtain that will aid diagnosis? 1.Rash on palms and feet 2.Shoulder and knee pain 3.Recent weight loss of 2 lb 4.Strep throat in the past 2 weeks

4. Strep throat in the past 2 weeks The smoky urine and the stated symptoms should lead the nurse to suspect glomerulonephritis, which usually occurs after a recent streptococcal infection. A rash on the hands and feet is associated with scarlet fever, not glomerulonephritis. Shoulder and knee pain is associated with rheumatic fever, not glomerulonephritis. Weight loss generally occurs in children who have type 1 diabetes, not those with glomerulonephritis

Which integumentary finding is related to skin texture? 1. Elasticity 2. Vascularity 3. Fluid buildup 4. Surface character

4. Surface character * Assessing for texture refers to evaluating the character of the surface of the skin. Assessing for elasticity refers to determining the turgor of the skin. Assessing for vascularity refers to determining skin circulation. Fluid buildup in the tissues indicated edema.

Which behaviors are observed in individuals at a conventional level of moral development? Select all that apply. A Obeying the rules B Respecting authority C Maintaining social order D Changing law in terms of societal needs E Orienting culturally to the labels of good or bad

A Obeying the rules B Respecting authority C Maintaining social order At the conventional stage, individuals are concerned with conformity and loyalty. Obeying the rules, doing one's duty, showing respect for authority, and maintaining the social order are the behaviors demonstrated during the conventional stage. Changing law in terms of societal needs is observed at the postconventional level. In the preconventional level, individuals are culturally oriented to the labels of good or bad and right or wrong. These labels of good and bad are then integrated into their concept of physical or pleasurable consequences of their actions.

The nurse who is caring for a child with botulism anticipates that the child's growth and development may be hampered due to cranial nerve deficits. Which signs presented by the child support the nurse's conclusion? Select all that apply. A Reduced gag reflex B Loss of head control C Accumulated secretions D Breathlessness in vocalizations E Paroxysmal muscle contractions

A Reduced gag reflex B Loss of head control Cranial nerve deficits are evidenced by diminished or reduced gag reflex or loss of head control. Accumulated secretions are seen due to laryngospasm and tetany of the respiratory muscles. Breathlessness in vocalizations is observed in Guillain-Barré syndrome due to intercostal and phrenic nerve involvement. Paroxysmal muscle contractions are observed in patients with tetanus due to extreme sensitivity to external stimuli.

A nurse is teaching a client how to use the call bell/call light system. Which level of Maslow's hierarchy of needs does this nursing action address? Safety Self-esteem Physiological Interpersonal

A call bell system enables the client to communicate with the staff and supports safety and security, which is a second-level need. Self-esteem involves intrapersonal needs, the fourth level of basic needs. Physiological needs include air, food, and water and represent the first level of needs. Interpersonal needs involve love and belonging, which are third-level needs.

Cracked and peeling skin Long scalp hair and fingernails Creases covering the neonate's full soles and palms

A client at 43 weeks' gestation has just given birth to an infant with typical postmaturity characteristics. Which postmature signs does the nurse identify? (Select all that apply.)

A nurse is examining different situations that represent the superego component of human personality, as per Sigmund Freud. Which situations accurately represent superego? Select all that apply.

A client controls the urge to eat candy because he or she knows that it will affect the blood sugar levels. A client having a craving for fruits does not steal them from the next client because that client needs it more. A client experiencing a stomachache refrains from stealing medications from a friend because it is illegal.

A nurse is evaluating different situations related to Maslow's hierarchy of needs. Which situations come under the second level of needs? Select all that apply.

A client tells the nurse that he or she is taunted by his or her boss every day. A client tells the nurse that his or her spouse belongs to a criminal gang. A client tells the nurse that he or she lives beside a factory that manufactures harmful chemicals.

Staying with her after bringing the infant to help her verbalize her feelings.

A health care provider tells a mother that her newborn has multiple visible birth defects. The mother seems composed and asks to see her baby. What nursing action will be most helpful in easing the mother's stress when she sees her child for the first time?

6 months

A mother is breastfeeding her newborn. She asks when she may switch the baby to a cup. The nurse concludes that the mother understands the teaching about feeding when she says she will start to introduce a cup after the baby reaches:

"The swelling and discharge are expected. They're a response to your hormones."

A mother is inspecting her newborn girl for the first time. The infant's breasts are edematous, and she has a pink vaginal discharge. How should the nurse respond when the mother asks what is wrong?

Cognitive Impairment

A neonate is tested for phenylketonuria (PKU) after formula feedings are initiated. The nurse explains to the parents that this is done to prevent:

30 to 60 breaths/min

A neonate weighing 5 lb 6 oz (2438 g) is born in a cesarean birth and admitted to the newborn nursery. What range of resting respiratory rate should the nurse anticipate?

"This often happens as the baby's head moves down the birth canal—the bones move for easier passage."

A new mother exclaims to the nurse, "My baby looks like a Conehead!" How should the nurse respond?

Start resuscitation

A newborn has an Apgar score of 3 at 1 minute after birth. What is the immediate nursing action in response to this Apgar score?

Rubella

A newborn has congenital cataracts, microcephaly, deafness, and cardiac anomalies. Which infection does the nurse suspect that the newborn's mother contracted during her pregnancy?

Milia

A newborn has small, whitish, pinpoint spots over the nose that are caused by retained sebaceous secretions. When documenting this observation, a nurse identifies them as:

3

A newborn of 30 weeks' gestation has a heart rate of 86 beats/min and slow, irregular respirations. The infant grimaces in response to suctioning, is cyanotic, and has flaccid muscle tone. What Apgar score should the nurse assign to this neonate?

Decrease the rate slowly

A newborn whose mother has type 1 diabetes is receiving a continuous infusion of fluids with glucose. What should the nurse do when preparing to discontinue the IV?

Notify the practitioner, because circumoral pallor may indicate cardiac problems

A newborn's hands and feet are cyanotic and there is circumoral pallor when the infant cries or feeds. What should the nurse do?

A demonstration and explanation of infant care

A nurse decides on a teaching plan for a new mother and her infant. What should the plan include?

Documenting the heart rate

A nurse determines that a 1-day-old newborn has a heart rate of 138 beats/min. What is the best nursing action at this time?

Cyanosis Tachypnea Retractions

A nurse determines that a newborn is in respiratory distress. Which signs confirm respiratory distress in the newborn? (Select all that apply.)

How to monitor their child for signs of jaundice

A nurse identifies a right cephalhematoma on an otherwise healthy 1-day-old newborn. What should the nurse teach the parents at the time of discharge?

It can be acquired during a vaginal birth.

A nurse in the newborn nursery receives a call from the emergency department saying that a woman with active herpes virus lesions gave birth in a taxicab while coming to the hospital. What does the nurse consider about the transmission of the herpes virus?

Jaundice that develops in the first 12 to 24 hours

A nurse is assessing a newborn for signs of hyperbilirubinemia (pathological jaundice). What clinical finding confirms this complication?

Ductus arteriosus Foramen ovale

A nurse is assessing a newborn with suspected retention of a fetal structure that will result in a congenital heart defect. Which fetal structures should undergo change after birth? (Select all that apply.)

A newborn's intestinal tract does not synthesize it for several days after birth.

A nurse prepares to administer vitamin K to a newborn. Why is vitamin K given specifically to newborns?

Adherence to a corrective diet instituted early

A nurse takes into consideration that the effect PKU has on the infant's development will depend on:

Infection

A nurse who is admitting a newborn to the nursery observes a fetal scalp monitor site on the scalp. For what complication should the nurse monitor this newborn?

Gastric acidity is low and does not provide bacteriostatic protection.

A parent of a newborn asks, "Why do I have to scrub my baby's formula bottles?" What information about a newborn should the nurse consider before replying in language that the parent will understand?

Lack the subcutaneous fat that usually provides insulation

A parent of a preterm infant in the neonatal intensive care unit, asks a nurse why the baby is in a bed with a radiant warmer. The nurse explains that preterm infants are at increased risk for hypothermia because they:

The nurse finds that a client with bilateral oral swelling, pain, and trismus had undergone a surgical extraction of an impacted tooth five days ago. What type of nursing diagnosis does the documentation of acute pain refer to?

Actual nursing diagnosis

A client presents to the emergency department with a fever, headache, loss of appetite, and malaise. The nurse identifies raised red bumps on the client's arms and legs. A diagnosis of chickenpox is made. The client should be placed in a private room with what kind of precautions?

Airborne precautions

Which delegation actions may be performed by unlicensed nursing personnel while caring for a client? Select all that apply.

Asking the client to wash hands before meals Instructing the client to wear footwear while walking

Which component is the ability to perform duties in a specific role?

Authority

A client receiving chemotherapy takes a steroid daily. The client has a white blood cell count of 12,000/mm3 and a red blood cell count of 4.5 million/mm3. What is the priority instruction that the nurse should teach the client?

Avoid large crowds and persons with infections

A nurse assisting in a research study calculates the risk-benefit ratio and concludes that there were no harmful effects associated with a survey of diabetic clients. This researcher was applying which principle?

Beneficence

The health care provider prescribes peak and trough levels of an antibiotic for a client who is receiving the medication intravenous piggyback (IVPB). For peak levels the nurse should have the laboratory obtain a blood sample from the client:

Between 30 and 60 minutes after the IVPB

A client is admitted with a suspected malignant melanoma on the arm. When performing the physical assessment, what would the nurse expect to find?

Brown or black mole with red, white, or blue areas

How does a nurse prepare a "factual" record when performing a client documentation?

By recording descriptive and objective information of what the nurse sees, hears, feels, and smells

How does a nurse incorporate the quality of accuracy into client documentation?

By using exact measurements for each client's activity

Which physical findings can be observed in a 2-month-old infant? Select all that apply. A The doll's eye reflex is fading. B The rooting reflex disappears. C The crawling reflex disappears. D The posterior fontanel is closed. E The primitive reflexes are fading.

C The crawling reflex disappears. D The posterior fontanel is closed. The crawling reflex disappears and the posterior fontanel is closed by 2 months of age. The doll's eye reflex disappears at 1 month of age. The primitive reflexes fade during the third month. The rooting reflex disappears at 4 months of age.

Which nursing process would the nurse undertake when collecting the medical history of a client? 1. Diagnosis 2. Evaluation 3. Assessment 4. Implementation

C. Assessment *The documentation of the client's information is part of an assessment. The nurse will collect all the relevant medical data of the client to help the health care provider understand the client's history and make an accurate diagnosis. During diagnosis, the collected data is analyzed to find out the clients problems or issues. Evaluation is the process to see if the expected outcomes of the treatment are achieved or not. Before an evaluation, a plan is made to solve all the client's problems and then the plan is implemented.

Type of care for a nurse providing care to one patient the entire shift

Case management

During the delegation process, which healthcare professional acts as a liaison between the team leader and healthcare provider?

Charge nurse

Which healthcare team member is responsible for the coordination and assignments of client care?

Charge nurse

Which nursing interventions require a nurse to wear gloves? -Giving a back rub -Cleaning a newborn immediately after delivery -Emptying a portable wound drainage system -Interviewing a client in the emergency department -Obtaining the blood pressure of a client who is human immunodeficiency virus (HIV) positive

Cleaning a newborn immediately after delivery and emptying a portable wound drainage system

The nurse is caring for four clients in an emergency department. Which client should be given least priority by the primary healthcare provider based on his/her condition?

Client with closed extremity trauma

A hospital needs to hire a nursing staff for the intensive care of cancer clients. Which of these positions is most likely to be filled by the nurse?

Clinical nurse specialist

A student nurse is learning Gardner's Tasks of Leading and Managing. Which position of the health care team has the managing function of priority setting, according to Gardner?

Clinical position

Which are considered as nurse competencies within the synergy model of care delivery? Select all that apply.

Collaboration clinical judgement cultural competency

A client who has participated in caring for her infant in the neonatal intensive care unit for several days in preparation for the infant's discharge comes to the unit on the last hospital day with an alcohol odor on her breath and slurred speech. What is the most appropriate action for the nurse to take at this juncture? Speak with the mother about her condition and assess her willingness to participate in an alternate discharge plan. Request that the mother wait in the hospital lobby and call the primary healthcare provider to cancel the discharge order. Speak to the mother about her condition and have her see a social worker about the infant's discharge to a foster home. Continue with the discharge procedure and alert the home health nurse that the mother needs an immediate follow-up visit.

Confrontation regarding the active substance abuse and the mother's diminished ability to care for the infant safely at this time is necessary to help the mother obtain help and to protect the infant. Decisions should not be made without input from the mother. Continuing with the discharge procedure and alerting the home health nurse that the mother needs an immediate follow-up visit is unsafe; the mother may not be capable of caring for the infant.

The nurse is explaining which developmental milestones a child's parents should expect to see at the age of 18 months. Which statements made by the parents indicate the need for further instruction? Select all that apply. 1 "My child will be able to take a few steps on tiptoe." 2 "My child will be able to pull and push toys." 3 "My child will be able to climb stairs with assistance." 4 "My child will be able to pick up objects without falling." 5 "My child will be able to kick a ball without losing balance."

Correct 1 "My child will be able to take a few steps on tiptoe." Correct 4 "My child will be able to pick up objects without falling." Correct 5 "My child will be able to kick a ball without losing balance." The child starts taking a few steps on tiptoe at the age of 30 months, not 18 months. The child will be able to pick up objects without falling and kick a ball forward at the age of 24 months, not 18 months. The gross motor skills of an 18-month-old child are well developed, so the child can pull or push toys. The child should also be able to climb stairs with one hand held.

The nurse is about to perform a wound irrigation on a client who had a left hemispheric stroke 1 year ago. Which assessment is most important for the nurse to perform before beginning the irrigation? Tachycardia Hypotension Rigid abdomen Nausea and vomiting Back and shoulder pain

Correct1 Tachycardia Correct2 Hypotension Correct3 Rigid abdomen Correct4 Nausea and vomiting Correct5 Back and shoulder pain

A registered nurse advises parents to assist their child with stretching exercises. The child has impaired physical mobility due to neuromuscular impairment. What is the rationale for performing stretching exercises? 1 To minimize pain 2 To prevent contractures 3 To promote the achievement of developmental milestones 4 To facilitate the mobilization of foods and fluids through the esophagus

Correct2 To prevent contractures A contracture deformity is the result of stiffness or constriction in the muscles. A contracture would adversely affect a child's development, depending upon location and severity. For example, if an infant developed a contracted Achilles tendon, the infant would not be able to physically develop the ability to walk. Administering pain medications helps decrease the pain. To promote the achievement of developmental milestones, the nurse encourages play exercises that involve joint movement and enhance fine and gross motor skill acquisition. The nurse places the child in the semi-upright position during feedings to facilitate the mobilization of food and fluids through the esophagus.

When assessing the cognitive development of a 9-year-old child, which characteristic indicates inadequate cognitive development? 1 The child collects different-colored rocks. 2 The child says that fall is better than spring. 3 The child considers a boy at school to be a better friend than a neighbor. 4 The child believes there are more blocks when spread out on the floor than in the container.

Correct4 The child believes there are more blocks when spread out on the floor than in the container. At ages 5 to 7, children learn that simply altering the arrangement of objects in space does not change certain properties of the objects. A 9-year-old child should able to resist perceptual cues that there are more blocks when on the floor than when in their container. During Piaget's stage of concrete operations, children develop an understanding of relationships between things and ideas. They become occupied with collections of objects, such as rocks, and derive enjoyment from classifying and ordering their environment. They may even begin to order friends and relationships (e.g., best friend, second-best friend).

Which medication is used in the treatment of a client with intervertebral disc disease?

Cyclobenzaprine

A client developed acute herpes zoster and was treated with antiviral medication within 72 hours of the appearance of the rash. The client is reporting persistent pain one week later. What does the nurse identify as the cause of the post therapeutic neuralgia?

Damage to the nerves

Which process involves transferring responsibility to multiple players, usually with varying degrees of education and experience, while retaining the ultimate accountability for providing the client care?

Delegation

A registered nurse is educating a nursing student about descriptive theories. Which point stated by the nursing student needs correction?

Descriptive theories help direct specific nursing activities.

Which description of accountability is correct?

Determination of appropriate actions and provide detailed explanations of the actions

What is the status of the unit secretary as a member of the healthcare team, which is in the span of control of a registered nurse (RN)?

Devoid of legal authority

When assessing the oral cavity of a newly admitted client with acquired immunodeficiency syndrome (AIDS), the nurse identifies areas of white plaque on the client's tongue and palate. What is the nurse's initial response?

Document the presence of the lesions, describing their size, location, and color

The nurse is caring for a client admitted with fluid overload. Which tasks are most appropriate to be delegated to the patient care associate? Select all that apply.

Documenting vital signs, documenting urine output Repositioning the client every one or two hours

What are the three strategies that the nurse can perform while assisting other nurses in making delegation decisions? Select all that apply.

Doing Asking Offering

Early parenting behavior

During a male newborn's first encounter with his mother the nurse encourages her to undress him. The mother strokes him with her whole hand and while looking at him intently says, "He feels so velvety, and he is going to be just as good looking as his daddy." The baby is alert and responsive while gazing at his mother. What is the nurse's assessment of this first mother-infant encounter?

Three vessels: one vein and two arteries

During a newborn assessment the nurse counts the infant's cord vessels. What does the nurse expect to observe in a healthy newborn?

Remove secretions from the pharynx

During the second reactive period a newborn becomes more alert and responsive and there is an increase in mucus production and gagging. What should the nurse do first?

What should the team leader identify as a priority when assigning clients to team members?

Each team member's strengths

The nurse needs to collect health care data for all employees at a multi- specialty hospital for research purposes. What type of health information technology would the nurse access to obtain the needed information?

Electronic health record (EHR)

Which nursing interventions are examples of the nurse as a caregiver? Select all that apply.

Encouraging the client to exercise daily Setting goals for the client to reduce weight Arranging for the client to meet a spiritual advisor

While assessing the client's skin, a nurse notices a skin condition, the pathophysiology of which involves increased visibility of oxyhemoglobin caused by an increased blood flow due to capillary dilation. Which condition is associated with this client?

Erythema

A case manager telephones a client with lung disease to evaluate respiratory function with home oxygen use and then contacts the primary care provider. What is the purpose of this communication?

Evaluate expected outcomes

Which statement best describes a fundamental aspect of the clinical nurse leader (CNL)?

Evaluates care for evidence-based approaches

While teaching a nursing student, a registered nurse says "This is a study in which the investigator controls the study variable and randomly assigns subjects to different conditions to test the variable." Which type of research is the above statement?

Experimental research

The nurse is caring for a client who is recovering from a stroke. The primary health care provider has referred the client for rehabilitative care. Which interventions by the nurse help to make a successful referral process? Select all that apply.

Explain the need for referral to the client and family. Provide the referral with adequate client information. Determine what the referral recommends for client care.

A primary health care provider prescribes airborne precautions for a client with tuberculosis. After being taught about the details of airborne precautions, the client is seen walking down the hall to get a glass of juice from the kitchen. The most effective nursing intervention is to:

Explore what the precautions mean to the client

A nurse is heading a performance improvement team. The nurse collects records of needlestick injuries due to improper needle recapping techniques used during medicine administration. What is the next step to be followed by the nurse, if he or she is using the "plan, do, study, act model

Facilitate a training program for all nurses to teach the proper technique of recapping needles

Which activity indicates improper follow-through on the part of the delegatee?

Failure to report results

A student nurse is assessing the blood pressure of a client with the client's arm unsupported. What are the expected errors in the obtained readings?

False high reading

A nurse is concerned about the public health implications of gonorrhea diagnosed in a 16-year-old adolescent. Which should be of most concern to the nurse?

Finding the client's contacts

The parents of a toddler who has been admitted to the pediatric unit for surgery to correct hypospadias ask the nurse when this defect happened. Which fetal development time period would the nurse respond? 1 First 12 weeks 2 Third trimester 3 Second 16 weeks 4 Implantation phase

First 12 Weeks

5

Five minutes after being born, a newborn is pale; has irregular, slow respirations; has a heart rate of 120 beats/min; displays minimal flexion of the extremities; and has minimal reflex responses. What is this newborn's Apgar score?

"What is the baby's daily schedule?"

Four weeks after giving birth, a client is agitated and tells the clinic nurse, "The baby cries all the time, and I don't know what to do." What question should the nurse ask before planning nursing care?

4 Agonal breaths

Gasping

A client who has been diagnosed with Lyme disease is started on doxycycline (Vibramycin) as part of the therapy. What should the nurse do when administering this drug?

Give the medication an hour before milk products are ingested.

A nurse is caring for a client with a diagnosis of acquired immunodeficiency syndrome (AIDS). The IV infiltrates and needs to be restarted. What is necessary to protect the nurse when restarting the IV? (Select all that apply.)

Gloves Hand hygiene

A client who abused intravenous drugs was diagnosed with the human immunodeficiency virus (HIV) several years ago. The nurse explains that the diagnostic criterion for acquired immunodeficiency syndrome (AIDS) has been met when the client:

Has a CD4+ T lymphocyte level of less than 200 cells/µL

While assessing a client, the nurse finds bluish coloration of the skin. The nurse finds that this discoloration is due to cyanosis. Which condition may be suspected?

Heart disease

A client complains of difficulty breathing. The nurse auscultates wheezing in the anterior bilateral upper lobes. What could be the possible reason for this sound?

High velocity airflow through an obstructed airway

Testing heel blood with the use of a glucose-oxidase strip

How should a nurse screen the newborn of a diabetic mother for hypoglycemia?

A nurse is reviewing the physical examination and laboratory tests of a client with malaria. For which important clinical indicators should the nurse be alert when reviewing data about this client? (Select all that apply.)

Hyperthermia Splenomegaly

A mother with the diagnosis of acquired immunodeficiency disease (AIDS) states that she has been caring for her baby even though she has not been feeling well. What important information should the nurse determine?

If the baby is breastfeeding

During an acquired immunodeficiency syndrome (AIDS) education class a client states, "Vaseline works great when I use condoms." Which conclusion about the client's knowledge of condom use can the nurse draw from this statement?

Ignorance related to correct condom use

How is a SOAP progress note different from a PIE progress note? Select all that apply.

In a SOAP progress note, the P stands for plan; in a PIE progress note, the P stands for problem. SOAP progress notes originate from medical records; PIE progress notes have a nursing origin. SOAP progress notes include assessment information; PIE progress notes do not include assessment information

The nurse uses evidence-based practice while providing nursing care to clients. What distinguishes research-based practice different from evidence-based practice?

It uses knowledge based only on research studies.

The physiological destruction of fetal red blood cells

Jaundice develops in a newborn 72 hours after birth. What should the nurse tell the parents is the probable cause of the jaundice?

What clinical manifestations does a nurse expect a client with systemic lupus erythematosus (SLE) most likely to exhibit? (Select all that apply.)

Joint pain Facial rash Pericarditis

A nurse is developing a teaching plan for a client with scleroderma. What should the nurse include about skin care?

Keep skin lubricated with lotion

What kind of health service does the nurse offer in a health promotion or primary care program?

Nutrition counseling

A client scheduled for surgery has a history of methicillin-resistant Staphylococcus aureus (MRSA) since developing an infection in a surgical site nine months ago. The site is healed and the client reports having received antibiotics for the infection. What should the nurse do to determine if the infecting organism is still present?

Obtain a prescription to culture the client's blood

Which steps are taken by a nurse during the implementation phase of drug research? Select all that apply.

Obtaining necessary approvals Recruiting subjects and collecting data obtained from the study

Which important intervention would be included in the nursing care provided immediately after a sexual assault? 1. Obtaining the assault history from the client 2. Reporting the assault to the police before the client is examined 3 Having the client void a clean-catch urine specimen 4 Testing the client's urine for seminal alkaline phosphatase

Obtaining the assault history from the client

What criteria should the nurse consider when determining if an infection should be categorized as a health care-associated infection?

Occurred in conjunction with treatment for an illness

An older client with a history of congestive heart failure expresses concern about potential exposure to tuberculosis. The client states that a roommate at the extended care facility where the client resides sleeps a lot, coughs a great deal, and sometimes spits up blood. The primary reason that the nurse pursues more information about the roommate is because:

Older adults with chronic illness are affected adversely by tuberculosis

A chronically ill, older client tells the home care nurse that the daughter with whom the client lives seems run-down and disinterested in her own health, as well as the health of her children, who are 5, 7, and 12 years old. The client tells the nurse that the daughter coughs a good deal and sleeps a lot. Why is it important that the nurse pursue the daughter's condition for potential case finding?

Older adults with chronic illness are more susceptible to tuberculosis

"The test won't be done until your baby has had enough milk for the results to be accurate."

On her first postpartum day, a client asks the nurse whether her baby has had a test for phenylketonuria (PKU) yet. How should the nurse reply?

Which sites would the nurse prefer while assessing for turgor in an older adult? Select all that apply.

On the sternal area & back of the fore arm

The nurse is caring for a client who underwent a hysterectomy and who is admitted to a general medical-surgical unit. Which tasks can be delegated to the unlicensed assistive personnel (UAP)? Select all that apply.

Oral hygeine; assistance with bathing

Which nursing theory focuses on the client's self-care needs?

Orem's theory

Which client assessment finding should the nurse document as subjective data?

Pain rating of 5

A client is admitted to the hospital for general paresis as a complication of syphilis. Which therapy should the nurse anticipate will most likely be prescribed for this client?

Penicillin therapy

Which assessing technique involves tapping a client's skin with the fingertips to cause vibrations in the underlying tissues?

Percussion

PPE -

Personal Protective Equipment

Which should the nurse include information for at each health maintenance visit for the parents of a toddler-age client?

Plants are the number one cause of accidental poisonings for the toddler-age client; therefore, the nurse should provide education related to this type of accidental poisoning at each health maintenance visit. Iron, aspirin, and corrosives also lead to accidental poisoning; however, these are not as common as plant poisoning.

Which are disadvantages to the functional system of care delivery? Select all that apply.

Poor communication Fragmentation of care changes in client status go unnoticed

The healthcare provider prescribes a low-fat, 2-gram sodium diet for a client with hypertension. The nurse should explain that the purpose of restricting sodium is to do what?

Prevent reabsorption of water in the distal tubules

What would the nurse state is the primary priority for decreasing a client's risk of morbidity and mortality?

Prevention

In the orientation phase, a nurse and a client meet and get to know each other. Which actions should the nurse follow in this phase? -Prioritizing the client's problems -Clarifying the client's and nurse's roles -Evaluating goal achievement with the client -Taking action to meet goals set with the client -Encouraging and helping the client with self-exploration

Prioritizing the client's problems and clarifying the client's and nurse's roles

Which position is indicated to assess the musculoskeletal system and is contraindicated in clients with respiratory difficulties?

Prone position Rationale: Prone position is indicated to assess the musculoskeletal system in clients, but it is indicated with caution in clients with respiratory difficulties because they cannot tolerate this position well. Sims position is indicated to assess the rectum and vagina. Supine position is indicated for general examination of head and neck, anterior thorax, breast, axilla, and pulses. Knee-chest position is indicated for rectal assessment.

What is the duty of the nurse as delegator if the delegatee has limited knowledge and ability to perform a task?

Provide guidance

A nursing student is listing the goals of theoretical nursing models. Which goal listed by the nursing student needs correction?

Provide knowledge to validate nursing interventions

How is public health nursing different from community health nursing?

Public health nursing focuses on population.

The student nurse is performing a rapid baseline assessment using a disability mnemonic (AVPU) in a client with drug abuse. Which parameters should the student nurse consider for proper assessment? Select all that apply.

Reaction to pain Response to voice

Which actions contribute to the transmission of human immunodeficiency virus (HIV) infection from an infected to a healthy person? Select all that apply.

Receiving blood transfusions Having sexual intercourse

A client with dehydration is prescribed an intravenous (IV) fluid infusion. Which healthcare professional would the nurse expect to be delegated this task?

Registered nurse

The client is experiencing postoperative pain and requests a pain shot. Which of the following healthcare providers are legally permitted to administer an intramuscular (IM) injection to the client? Select all that apply.

Registered nurse (RN) Licensed practical nurse (LPN) Licensed vocational nurse (LVN)

A client arrives for a vaccination at an influenza prevention clinic. A nursing assessment identifies a current febrile illness with a cough. The nurse should:

Reschedule administration of the vaccine for the next month

An increased Paco 2 of 55 mm Hg

Respiratory acidosis is confirmed in a neonate with respiratory distress syndrome when the laboratory report reveals:

A client is admitted with the diagnosis of tetanus. For which clinical indicators should the nurse assess the client? (Select all that apply.)

Restlessness Muscular rigidity Respiratory tract spasms Spastic voluntary muscle contractions

A nurse observes that an unlicensed assistive personnel (UAP) did not use a bag impervious to liquid for contaminated linen from a client who is on contact precautions. The nurse's best way to handle this situation is to:

Review transmission-based precautions with the UAP

A reasonable short-term outcome for clients who are functioning below the optimal level of mental health is to help them become better able to do what? Understand the dynamics behind their inadequate interpersonal relations. Confront their inadequacies in interpersonal relations and be more sociable. Discuss feelings regarding their life experiences and their significant others. Take actions that will increase their satisfaction with their relationships with others.

The ability to discuss feelings about others and life situations is necessary for positive mental health. Understanding interpersonal dynamics, confronting inadequacies, and taking actions to increase satisfaction in relationships are all long-term, not short-term, outcomes.

According to Piaget's theory, what are the cognitive or moral developmental changes in children aged 6 to 12 years? Select all that apply.

The child develops logical thinking. The child is in the concrete operations period

The parents of a 6-year-old boy tell the nurse in the pediatric clinic that their son has recently started to wet the bed at night. What is the most helpful response by the nurse? "How's your son doing in school?" "Have there been any changes in his life recently?" "You should arrange to see the doctor, because there may be a physical problem." "When children are angry at their parents, they may use bed-wetting to punish them."

The collection of more information is essential before the nurse can intervene further. Asking a general question opens the lines of communication. Asking specifically about school might be appropriate later in the discussion. Suggesting a medical consultation is premature; more information is needed. Enuresis is usually not a behavioral response precipitated by anger; this statement may cause the parents to feel guilty or become defensive.

"This is the time when the baby is likely to be most responsive to you."

The nurse is caring for a couple after the birth of their first child. What should the nurse tell the family to do when their infant is exhibiting the behavior demonstrated in the picture?

The cause is an increased intravascular pressure during birth.

The parents of a newborn are concerned about red pinpoint dots on their infant's face and neck. How should the nurse explain the finding?

The nurse leader is giving a speech on leadership skills to followers. Which questions enable the nurse leader to evaluate the understanding level of the followers? Select all that apply. "Are you getting my points?" "Would you all like a break?" "Can I change the topic in a little while?" "How can you solve a conflict at the workplace?" "What did you 'hear' in the process of this communication?"

The question such as "what did you 'hear' in the process of this communication?" cannot be answered by a "yes" or "no" and requires a detailed explanation of the things that are taught. The questions starting with "how" are usually open-ended and require the person to answer in detail. Therefore these questions can help the nurse leader evaluate the understanding level of the followers. The questions such as "are you getting my points?", "would you all like a break?", and "can I change the topic in a little while?" can be answered by a "yes" and "no." Therefore these questions do not help the nurse leader to evaluate the understanding level of the followers.

A client is admitted to the hospital after having a tonic-clonic seizure. The client has a two-year history of a seizure disorder, but the seizures have been well controlled by phenytoin for the last six months. The client says to the nurse, "I am so upset. I didn't think I was going to have more seizures." Which is the best response by the nurse? "Did you forget to take your medication?" "You are worried about having more seizures?" "You must be under a lot of stress right now." "Don't be too concerned

The response "You are worried about having more seizures?" addresses the client's feelings and encourages communication. The question "Did you forget to take your medication?" sounds accusatory; it ignores the client's feelings and discourages communication. Although the statement "You must be under a lot of stress right now" may be true, it does not encourage further communication concerning the seizure. The statement "Don't be too concerned because your medication needs to be increased" negates the client's feelings and discourages communication.

A nurse stops by the room of a newly admitted depressed client and offers to walk with the tearful client to the evening meal. The client looks intently at the nurse but says nothing. What is the best response by the nurse? "I'll be at the desk if you need me." "You must tell me what you're feeling now." "We'll walk together to dinner when you calm down." "It must be very difficult for you to be on a psychiatric unit."

The statement "It must be very difficult for you to be on a psychiatric unit" lets the client know that the nurse realizes that the client is having difficulty without asking direct questions or focusing on specific behavior. The response "I'll be at the desk if you need me" connotes avoidance. Saying "You must tell me what you're feeling now" sounds more like an order than an opportunity to express feelings. Saying "We'll walk together to dinner when you calm down" negates the client's feelings. The nurse should talk to the client without any expectation that the client will "calm down."

As depression begins to lift, a client is asked to join a small discussion group that meets every evening on the unit. The client is reluctant to join and says, "I have nothing to talk about." What is the best response by the nurse? "Maybe tomorrow you'll feel more like talking." "Could you start off by talking about your family?" "A person like you has a great deal to offer the group." "You feel you won't be accepted unless you have something to say?"

The statement about the client's feelings of acceptance is a reflective statement that allows the client to either validate the statement or correct the nurse. Postponing the conversation delays addressing the problem and avoids exploring feelings. Asking the client to start talking about her or his family is a response that gives advice and does not allow the client to explore feelings. Stating that the client has a lot to offer the group denies the client's statement and does not allow the exploration of feelings.

The nurse as a leader provides feedback to a newly recruited nursing student after checking the student's progress report. Which action of the registered nurse is most closely aligned with the application of two-factor theory during the feedback session? Creating enthusiasm for practice Ignoring negative behaviors of the student nurse Promoting job enrichment by creating job satisfaction Providing specific feedback about positive performance

The two-factor theory of leadership indicates that motivating factors such as promoting job enrichment by creating job satisfaction inspire the work performance of the staff. Creating enthusiasm for staff practice characterizes the transformational theory of leadership. Ignoring the negative behaviors of student nurses indicates an application of the Organizational Behavior Modification theory of leadership. Providing specific feedback about positive performance indicates the application of the Expectancy theory of leadership.

A nurse working in the health services center of a college is reviewing the vaccination records of a young adult who plans to enroll. Which immunizations are required to meet admission criteria according to the American Academy of Pediatrics?

Three doses of diphtheria toxoid and oral poliomyelitis vaccine, and one dose of live measles, live rubella, and mumps vaccine

A nurse working in the health services center of a college is reviewing the vaccination records of a young adult who plans to enroll. Which immunizations are required to meet admission criteria according to the American Academy of Pediatrics?

Three doses of diphtheria toxoid and oral poliomyelitis vaccine, and one dose of live measles, live rubella, and mumps vaccine.

A client cannot understand how syphilis was contracted because there has been no sexual activity for several days. Which length of time associated with the incubation of syphilis should the nurse include in the teaching plan?

Two to six weeks

The nurse teaches a group of clients that nutritional support of natural defense mechanisms indicates the need for a diet high in:

Vitamins A, C, E, and selenium

A nurse is planning to provide discharge teaching to the family of a client with acquired immunodeficiency syndrome (AIDS). Which statement should the nurse include in the teaching plan?

Wash used dishes in hot, soapy water."

The nurse is advising a client to carry a prescription of epinephrine autoinjector. Which insect bite or sting is responsible for the nurse providing this advice?

Wasp

A nurse is caring for a client who has been taking several antibiotic medications for a prolonged time. Because long-term use of antibiotics interferes with the absorption of fat, the nurse anticipates a prescription for:

Water-soluble forms of vitamins A and E

Which statement is applicable to Watson's theory of transpersonal caring?

Watson's theory defines the outcome of nursing activity in relation to the humanistic aspects of life.

Barely visible areolae and nipples

What characteristic does the nurse anticipate in an infant born at 32 weeks' gestation?

Protruding tongue Epicanthal eye folds One transverse palmar crease

What characteristics cause the nurse to suspect that a newborn has Down syndrome? (Select all that apply.)

Protruding tongue Hypotonic muscle tone Broad nose with a depressed bridge

What clinical findings does the nurse expect to observe in a newborn with trisomy 21 (Down syndrome)? (Select all that apply.)

Protecting the skin surrounding the exposed bladder

What should be included in the teaching plan for the mother of a newborn with exstrophy of the bladder?

Learning specific behaviors involving states of wakefulness to promote positive interactions

What should the nurse discuss with new parents to help them prepare for infant care?

Help the parents stimulate their awake baby through touch, sound, and sight

What should the nurse do to enhance a neonate's behavioral development?

Use tactile stimuli on the chest or extremities

What should the nurse do when an apnea monitor sounds an alarm 10 seconds after cessation of respirations?

Having the visitor step outside the room

When a nurse who is carrying a newborn to the mother enters the room, a visitor asks to hold the infant. The visitor is sneezing and coughing. What is the most important measure for the nurse to take?

Small breast buds Wrinkled thin skin Pinnae that remain flat when folded

Which characteristics should alert the nurse to conclude that a male newborn is a preterm infant? (Select all that apply.)

Face Trunk Buttocks

Which parts of a newborn's body are usually affected by the rash erythema toxicum neonatorum? Select all that apply.

"Infants' feet appear flat because the arch is covered with a fat pad."

While inspecting her newborn a mother asks the nurse whether her baby has flat feet. How should the nurse respond?

Which figure indicates that a nurse is performing a palpation of the popliteal pulse?

[behind the knee]

A 7-month-old infant is admitted to the hospital with a diagnosis of acute gastroenteritis. What will be the nursess priority goal of the infants care? a. Prevent fluid and electrolyte imbalance. b. Prevent nutritional deficiency. c. Prevent skin breakdown. d. Prevent malabsorption.

a

What instruction will the nurse give to parents about preventing the spread and reinfection of pinworms? a. Keep childrens nails short. b. Dress child in loose-fitting underwear. c. Clean the bathroom with bleach solution. d. Wash bed linens in cold water.

a

What is the treatment of choice for a child with intussusception? a. A barium enema b. Immediate surgery c. IV fluids until the spasms subside d. Gastric lavage

a

Which assessment would the nurse report to the physician immediately? a. 2-month-old with a urine output of 150 mL in 24 hours b. 3-year-old with a urine output of 650 mL in 24 hours c. 8-year-old with a urine output of over 1000 mL in 24 hours d. 14-year-old with a urine output of 800 mL in 24 hourse

a

Which is the most appropriate intervention for a 3-month-old infant who has gastroesophageal reflux? a. Position the infant in the crib on his or her abdomen, with the head elevated. b. Administer medication as ordered to stimulate the pyloric sphincter. c. Give thin rice cereal with formula before feeding solid foods. d. Place the infant in an infant seat after feedings.

a

What sign(s) indicate(s) moderate dehydration? (Select all that apply.) a. 10% weight loss b. Dry mucous membranes c. Normal anterior fontanel d. Increased urinary output e. Lethargy

a, b, c

What assessment(s) would lead a nurse to suspect Hirschsprungs disease in a 1-month-old infant? (Select all that apply.) a. Ribbon-like stools b. Fever c. Failure to thrive d. Vomiting e. Diminished peristalsis

a, b, c, d, e

Parents ask the nurse how their infant developed a Meckels diverticulum. What condition, will the nurse explain, is present causing this diagnosis? a. The yolk sac remains connected to the intestine. b. There is inflammation of the ileocecal valve. c. A pouch forms when the vitelline duct fails to disappear. d. There is a weakness in the abdominal wall.

c

The nurse is planning a parent education program about lead poisoning prevention. What will be included regarding primary sources of lead in the community? a. Increased lead content of air b. Use of aluminum cookware c. Deteriorating paint in older buildings d. Inhaling smog

c

The nurse is speaking to the parent of a 3-year-old child who has mild diarrhea. What dietary modification would the nurse advise? a. Soft foods with rice, bananas, toast, and applesauce b. Small amounts of clear fluids such as gelatin c. An oral rehydrating solution, such as Pedialyte d. Chicken soup because it is high in sodium

c

The nurse is teaching a parent about pyrvinium (Povan). What would be included in regard to potential side effects? a. Diarrhea b. Skin rash c. Red stool d. Metallic taste

c

Why are infants more vulnerable to fluid and electrolyte imbalances than adults? a. They have a smaller surface area than adults in proportion to body weight. b. Water needs and losses per kilogram are lower than those for adults. c. A greater percentage of body water in infants is extracellular. d. Infants have a lower metabolic turnover of water.

c

Why are rapid respirations a possible cause of dehydration? a. They prevent the child from drinking. b. They increase circulation, thus increasing urine production. c. They cause evaporation of fluid on the mucous membranes. d. They often lead to vomiting.

c

Following surgery for pyloric stenosis, an infant awoke from anesthesia hungry and crying. What is the most appropriate nursing action? a. Delay feeding the child for 6 hours. b. Offer regular formula thinned with water. c. Give small amounts of regular formula thickened with cereal. d. Allow 1 ounce of glucose water at frequent intervals.

d

The nurse has reviewed dietary restrictions for celiac disease with concerned parents. Which grain will the nurse explain can be eaten with celiac disease? a. Wheat b. Oats c. Barley d. Rice

d

The nurse is caring for an 18-pound child who has had one stool of diarrhea. The nurse knows that the child needs to consume how many milliliters of oral fluid to make up for the fluid loss? a. 18 b. 36 c. 64 d. 81

d

The nurse is instructing a mother how to administer oral nystatin suspension prescribed to treat thrush. What will the nurse include? a. Pour the prescribed amount into a nipple and have the infant suck the medication. b. Squirt the prescribed dose into the back of the mouth and have the infant swallow. c. Give the medication mixed with a small amount of juice in a bottle. d. Use a sterile applicator to swab the medication on the oral mucosa.

d

Dyspnea

difficult or labored breathing

dysgeusia

distortion of the sense of taste

antihypertensive

drug that lowers blood pressure

xerostomia

dryness of the mouth

A client with a head injury underwent a physical examination. The nurse observes that the client's temperature assessments do not correspond with the client's condition. An injury to which part of the brain may be the reason for this condition?

hypothalamus

A female client is upset with her diagnosis of gonorrhea and asks the nurse, "What can I do to prevent getting another infection in the future?" The nurse evaluates that the teaching is understood when the client states, "My best protection is to:

insist that my partner use a condom."

Which step of the nursing process is directly affected if the nurse does not make a nursing diagnosis?

planning

The nurse pulls up on the client's skin and releases it to determine whether the skin returns immediately to its original position. What is the nurse assessing for?

skin turgor

Influenza epidemic take orders from

state officials

Subjective data

things a person tells you about that you cannot observe through your senses; symptoms

Which parts of the body should be assessed for temperature in clients who abuse sedatives or hypnotics? Select all that apply.

thorax & forehead

Debillitated

weakened

A nurse is recalling Piaget's theory of cognitive development. What is the characteristic of the preoperational stage?

"An infant may learn to think with the use of symbols and mental images."

A nursing student is listing the steps that need to be followed for applying developmental theory when caring for chronically ill older adults with depression. Which step listed by the nursing student needs correction?

"The nurse should recognize the need to identify depression so that heart failure can be prevented."

An older adult with chills arrived to hospital. The nurse assesses the client's vital signs and determined the client has a fever. What would be the client's rectal temperature?

38.5

Has type 1 diabetes

A nurse expects signs of respiratory distress syndrome (RDS) in a neonate whose mother:

A client reports to the nurse sleeping until noon every day and taking frequent naps during the rest of the day. What should the nurse do initially?

Arrange a referral for a thorough medical evaluation.

A student nurse prepares a concept map while caring for a client recovering from surgery. What is the first step that the student nurse should take when preparing the concept map?

Arrange cues into clusters that form patterns

What is the order of evidence-based practice that a nurse should follow while caring for a client?

Ask a question Collect evidence Appraise the evidence Make a decision Evaluate changes Share outcomes

Which physiologic changes are observed in pregnant women during the third trimester? Select all that apply. A Fatigue B Morning sickness C Urinary frequency D Breast enlargement E Braxton Hicks contractions

Correct A Fatigue Correct C Urinary frequency Correct E Braxton Hicks contractions Due to the enhanced growth of the fetus and uterus in the third trimester, physiologic changes including fatigue, urinary frequency, and Braxton Hicks contractions are observed in pregnant women. Women in the first trimester may experience morning sickness, breast enlargement and tenderness, and fatigue.

A nurse is completing the Resident Assessment Instrument (RAI) for a resident at the nursing center. What should the nurse consider while completing the RAI? Select all that apply.

Minimum Data Set (MDS) Resident Assessment Protocols Utilization guidelines of the state

A perfect score is 10; 1 point is deducted for lessened muscle tone (the baby's arms do not flex) and 1 point for acrocyanosis, which is manifested by bluish hands and feet.

One minute after birth a nurse notes that a newborn is crying, has a heart rate of 140 beats/min, is acrocyanotic, resists the suction catheter, and keeps the arms extended. What Apgar score should the nurse assign to the newborn? Record your answer using a whole number. ___

Which member of the healthcare team would the nurse claim performs functions through passive delegation?

Pharmacist

Client A is recovering from heart surgery and needs to adapt to his or her environment. Client B is at the last stage of cancer. Which of these theories may the nurse use for better health maintenance of both the clients?

Roy's theory for client A and Henderson's theory for client B

An infant is admitted to the hospital with severe dehydration. Laboratory results show pH 7.32, PaCO2 40, HCO3 21. How does the nurse interpret these values? a. Metabolic acidosis b. Metabolic alkalosis c. Respiratory acidosis d. Respiratory alkalosis

a

On the second day of hospitalization for a 3-month-old brought in for treatment for gastroenteritis, the nurse makes all of the assessments listed below. Which assessment finding indicates ineffectiveness of treatment? a. Weight loss of 4 ounces b. Dry mucous membranes c. Decreased skin turgor d. Depressed fontanelle

a

Parents have adopted a child with the diagnosis of kwashiorkor. What is most likely to be observed when assessing this child? (Select all that apply.) a. Hyperactivity b. White streak in hair c. Edematous abdomen d. Slowed growth e. Thick, oily hair

b, c, d

A nurse is evaluating situations based on the responses of several clients. Which client's statement confirms that he or she has reached the Integrity versus Despair stage according to Erikson's theory of psychosocial development? Select all that apply.

"Looking back at my entire life, I find that I have actually achieved nothing." "In the twilight of my life, I regret not fulfilling the promises I made to my wife." "Now that I am at the end of the road, I think I am the luckiest person on the earth because God has given me everything that I had asked for."

A client with schizophrenia reports having ongoing auditory hallucinations and describes them as "voices telling me that I'm a bad person" to the nurse. What is the best response by the nurse? "Try to ignore the voices." "What are the voices saying to you?" "Do you believe what the voices are saying?" "They're only voices, so just try not to be afraid."

"Try to ignore the voices." Clients can sometimes learn to push auditory hallucinations aside, particularly within the framework of a trusting relationship; it may provide the client with a sense of power to manage the voices. Once it has been established by the nurse that the voices are not commanding the client to self-harm or harm others, focusing on the content of the hallucinations is not therapeutic. Asking whether the client believes what the voices are saying or encouraging the client not to be afraid of them is irrelevant to the situation; clients believe in and are frightened by hallucinations.

Which description by the nurse is a correct explanation of delegation?

***not accountability*** The transfer of responsibility for the performance of an activity

The nurse is conducting a client interview. Which response by the nurse is an example of back channeling? 1. "All right, go on..." 2. "What else is bothering you?" 3. "Tell me what brought you here.? 4. "How would you rate your pain on a scale of 0 to 10?"

1. "All right go on..." *Back channeling involves the use of active listen prompts such as "Go on...", "All right," and "uh-huh." Such prompts encourage the client to complete the full story. The nurse uses probing by asking the client, "What else is bothering you?" Such open-ended questions help obtain more information until the client has nothing more to say. The statement, "Tell me what brought you here" is an open-ended statement that allows the client to explain his or her health concerns in his or her own words. Closed-ended questions such as, "How would you rate your pain on a scale of 0 to 10?" are used to obtain a definite answer. The client answers by stating a number to describe the severity of pain.

A child is being treated with oral ampicillin (Omnipen) for otitis media. What should be included in the discharge instructions that the nurse provides to the parents of the client? 1. Complete the entire course of antibiotic therapy. 2. Herbal fever remedies are highly discouraged. 3.Administer the medication with meals. 4.Stop the antibiotic therapy when the child no longer has a fever.

1. Complete the entire course of antibiotic therapy Once antibiotics therapy is initiated, the antibiotics start to destroy specific bacterial infections that the health care provider is trying to treat. Antibiotic therapy takes a specific dose and number of days to completely eliminate the bacteria. If the caregivers start a dose and stop it before the course is complete, the remaining bacteria has a chance to grow again, become resistant to antibiotic treatment, and multiply. The nurse should not discourage use of herbal fever remedies; however the herbal treatment should be reviewed to see if it is contraindicated. Ampicillin should be taken 1 to 2 hours after meals. Antibiotic therapy should be completed as prescribed. View Topics

The client reports difficulty in breathing. The nurse auscultates lung sounds and assess the respiratory rate. Which is the purpose of the nurses' action? 1. Data collection 2. Data validation 3. Data clustering 4. Data interpretation

1. Data collection *The nurse is gathering objective data to support the subjective data. The client's report of difficulty breathing is subjective data that must be supported by data obtained during the physical examination. The nurse reviews the database after data collection to decide if it is accurate and complete. This step is called data validation. The grouping of data that forms a pattern is called data clusters. The nurse uses critical thinking to interpret the data and analyze it before it is classified and organized into data clusters.

According to Quality and Safety Education for Nurses (QSEN), which defines patient-centered care? 1. Understanding that the client is the source of control when providing care 2. Functioning effectively within nursing and interprofessional teams to deliver quality care 3. Using data to evaluate outcomes of care processes and designing methods to improve health care 4. Minimizing the risk for harm to clients and health care workers through improved professional performance.

1. Understanding that the client is the source of control when providing care * The QSEN competency called patient-centered care requires the nurse to understand that the client is the sources of control. The nurse would respect the values, beliefs, and preferences of the client to provide quality care. The QSEN competency called teamwork and collaboration states that the nurse would function effectively within nursing and interprofessional teams to provide quality care. Quality improvement involves using data to evaluate the outcomes of care processes and design methods to improves the health care delivery system. Safety focuses on minimizing the risk for harm to clients and health care workers through improved professional performance.

What should the nurse teach parents about their newborn's diagnosis of phenylketonuria (PKU)? 1.A low-phenylalanine diet is required. 2.Phenylalanine is not necessary for growth. 3.Phenylalanine can be administered to correct the deficiency. 4.A substitute for phenylalanine is an increased amount of other amino acids.

1.A low-phenylalanine diet is required Reducing dietary phenylalanine helps prevent brain damage. The PKU diet is planned to maintain the serum phenylalanine level at 2 to 8 mg/100 mL. Phenylalanine is essential for growth and development of the brain. Administering phenylalanine is contraindicated. There are no substitute for phenylalanine, which is one of the essential amino acids.

A 12-year-old child with Down syndrome is admitted to the hospital for intravenous antibiotics for pneumonia. Which clinical findings associated with Down syndrome should the nurse expect when performing a physical assessment? Select all that apply. 1. Saddle nose 2. Thin fingers 3. Inner epicanthic folds 4. Hypertonic musculature 5. Transverse palmar crease

1.Saddle Nose 2.Inner epicanthic folds 3.Transverse palmar crease Children with Down syndrome have a broad nose with a depressed bridge (saddle nose), as well as inner epicanthic folds, and oblique palpebral fissures; they also have speckling of the iris (Brushfield spots). Children with Down syndrome have a transverse palmar crease (simian crease) formed by fusion of the proximal and distal palmar creases. These children also have broad, short, stubby hands and feet. Children with Down syndrome have hypotonic, not hypertonic, musculature.

Which question should the nurse include in the assessment process for the parent of a toddler-age client who is diagnosed with lead toxicity that is not related to a household item? 1 "Do you teach for a living?" 2 "Do you refinish furniture?" 3 "Do you work in a hospital?" 4 "Do you work with children?"

2. "Do you refinish furniture?" Occupational exposure to lead can occur when a parent works with refinishing furniture; therefore, this is a question the nurse should include in the assessment process. Teaching, working in a hospital, and working with children are not occupational exposure risks for lead.

Which is appropriate for the nurse to include in the education of the ethical principle of nonmaleficence to a group of nursing students? 1. Treat all clients equitably and fairly 2. Act in ways to prevent harm to clients 3. Tell the client the truth about their health 4. Help the clients make informed choices

2. Act in ways to prevent harm to clients * Nonmaleficence means to act in ways that prevent client harm or even the risk of harm. Telling the truth to clients about their health refers to veracity. Helping clients make informed choices promotes autonomy. Justice involves treating all clients equitably and fairly.

Which group benefits from Medicare? 1. Self-insured employers 2. People who are 65 years or older 3. Members of low-income families 4. Children who are not poor enough for Medicaid

2. People who are 65 years or older *Medicare is a health insurance program for people 65 years or older. The payment for the plan is deducted from monthly individual Social Security checks. A preferred provider organization (PPO) plan is a contractual agreement between a set of providers and self-insured employers. It offers comprehensive health services at a discount to companies under contract. The Medicaid plan is a federally funded, state-run program that provides health insurance for low-income families. It finances a large portion of care for poor children, their parents, pregnant women, and disabled very poor adults. The State Children's Health Insurance Programs (SCHIP) is a federally funded, state-run program for children who are not poor enough for Medicaid.

Which cation regulates intracellular osmolarity? 1. Sodium 2. Potassium 3. Calcium 4. Calcitonin

2. Potassium * A decrease in serum potassium causes a decrease in the cell wall pressure gradient and results in water moving out of the cell. Besides intracellular osmolarity regulation, potassium also regulates metabolic activities, transmission and conduction of nerve impulses, cardiac conduction, and smooth and skeletal muscle contraction. Sodium is the most abundant extracellular cation that regulates serum osmolarity as well as nerve impulse transmission and acid-base balance. Calcium is an extracellular cation necessary for bone and teeth formation, blood clotting, hormone secretion, cardiac conduction, transmission of nerve impulses, and muscle contraction. Calcitonin is a hormone secreted by the thyroid gland and works opposite of parathormone to reduce serum calcium and keep calcium in the bones. Calcitonin does not have a direct effect on intracellular osmolarity.

The nurse is assessing a client after surgery. Which assessment finding would the nurse obtain from the primary source? 1. X-ray reports 2. Severity of pain 3. Results of blood work 4. Family caregiver interview

2. Severity of pain * The primary source of information during an assessment is the client. The nurse gathers information about the clients pain from the primary source, the client. Medical records such as X-ray reports and results of blood work are secondary sources of information. The clients family caregiver is secondary source of information.

A client asks the nurse, "Should I tell my partner that I just found out I'm human immunodeficiency virus (HIV) positive?" Which is the nurse's most appropriate response? 1 "Do not tell your partner unless asked." 2 "This is a decision you alone can make." 3 "You are having difficulty deciding what to say." 4 "Tell your partner that you don't know how you became sick."

3 "You are having difficulty deciding what to say."

An infant is being admitted with bacterial meningitis. The nurse knows the priority nursing action is: 1 Assessing the infant's neurological status 2 Beginning intravenous fluids and antibiotics 3 Implementing respiratory isolation precautions 4 Teaching the parents the importance of maintaining a quiet environment

3 Implementing respiratory isolation precautions The infant's illness is contagious, and the nurse, as well as other clients, must first be protected with the implementation of respiratory isolation precautions. Assessment of neurological status, implementation of prescribed fluids and antibiotics, and parental teaching may be done after assessment. Also, antibiotics are usually not administered until after all cultures have been obtained.

After collecting data on a 2-month-old infant, the nurse reinforces proper safety measures to the infant's mother to reduce the risk of injury in the infant. Which statements made by the infant's mother need correction? Select all that apply. 1 "I should use cool mist vaporizers if my baby has a cold." 2 "I should provide a one-piece pacifier to soothe my baby." 3 "I should protect my child's crib mattress with a plastic covering." 4 "I should place a firm mattress and loose blankets in my baby's bed." 5 "I should ensure that my child's car seat is rear-facing in a seat with an airbag."

3 "I should protect my child's crib mattress with a plastic covering." 5 "I should ensure that my child's car seat is rear-facing in a seat with an airbag." Covering a crib mattress with plastic should be avoided, as it can expose the child to toxins. The nurse should instruct the mother to refrain from placing the infant in a seat with an air bag, as it can cause suffocation and trauma. The nurse should inform the mother to use cool mist vaporizers, as they prevent burns. The mother should provide a one-piece pacifier to the infant, as it prevents accidental swallowing of any small objects and aspiration. Placing the infant in a crib with a firm mattress and loose blankets helps to prevent suffocation.

A 50-year-old client seen for a routine physical asks why a stool specimen for occult blood testing has been ordered. Which is the correct nursing response? 1. "You will need to ask your health care provider; it is not part of the usual tests for people your age." 2. "There must be concern of a family history of colon cancer; that is a primary reason for an occult blood stool test." 3. "It is performed routinely starting at your age as part of an assessment for colon cancer." 4. "There must have been a positive finding after a digital rectal examination performed by your health care provider."

3. "It is performed routinely starting at your age as part of an assessment for colon cancer." * The primary reason for a stool specimen for guaiac occult blood testing is that it is part of a routine examination for colon cancer in any client over the age of 40 years. Age, family history of polyps, and a positive finding after a digital rectal examination are factors related to colon cancer and secondary reasons for the occult blood test (guaiac test).

Which assessment is expected when a client is placed in lithotomy position during physical examination? 1. Heart 2. Rectum 3. Female genitalia 4. Musculoskeletal system

3. Female genitalia * Lithotomy position in female clients is used to assess and examine female genitalia and genital tracts. The lateral recumbent position is indicated in clients to assess the heart. The knee-chest position and Sims position are recommended for clients undergoing rectal examinations. The prone position is indicated in clients to assess the musculoskeletal system.

Which caring intervention helps provide comfrot, dignity, respect, and peace to a client? 1. Listening 2. Spiritual caring 3. Providing presence 4. Relieving pain and suffering

4. Relieving pain and suffering * Relieving pain and suffering is not just about giving medications but includes providing comfort, dignity, respect, and peace to a client. Listening helps obtain meaning interactions with clients. Spiritual caring helps clients find balance between their own life values, goals, and belief systems. Providing presence helps convey closeness and a sense of caring.

A nurse is caring for a 3-month-old infant with congenital hypothyroidism. What should the parents be taught about the probable effect of the condition on the infant's future if treatment is not begun immediately? 1 Myxedema 2 Thyrotoxicosis 3 Spastic paralysis 4 Cognitive impairment

4 Cognitive impairment Congenital hypothyroidism is the result of insufficient secretion by the thyroid gland because of an embryonic defect. A decreased level of thyroid hormone affects the fetus before birth during cerebral development, so it is likely that there will be some cognitive impairments at birth. Treatment before 3 months will prevent further damage. Congenital hypothyroidism does not become myxedema. Thyrotoxicosis is another term for hyperthyroidism. Although it is not expected, it may occur with an overdose of exogenous thyroid hormone, but it is too soon to discuss this possibility with the parents. Spastic paralysis occurs only if the infant has cerebral palsy.

A nurse is caring for an infant with phenylketonuria. What diet should the nurse anticipate will be prescribed by the health care provider? 1 Fat free 2 Protein-enriched 3 Phenylalanine-free 4 Low-phenylalanine

4 Low-phenylalanine Because phenylalanine is an essential amino acid it must be provided in quantities sufficient for the promotion growth but low enough to maintain a safe blood level. Phenylalanine is derived from protein, not fat. An enriched-protein diet contains increased amount of proteins, including phenylalanine, which should be ingested in limited amounts. Because phenylalanine is an essential amino acid, it cannot be totally removed from the diet.

According to Kohlberg's moral judgment theory, which characteristic behavior would the nurse find in the child who is in the naive instrumental orientation stage? 1 The child tries to follow laws and respects order. 2 The child tries to develop good social relationships. 3 The child follows the rules due to fear of punishment. 4 The child is motivated by a selfish desire to obtain rewards and benefits.

4 The child is motivated by a selfish desire to obtain rewards and benefits. According to Kohlberg's moral judgment theory, every child has a gradual development of moral consciousness based on cognitive development. During the naive instrumental orientation stage, the child's behavior is motivated by a selfish desire to obtain rewards and benefits. During the law and order orientation stage, the child tries to follow rules and laws. During the social contract orientation stage, the child tries to develop good social relationships. During the punishment and obedience orientation stage, the child obeys rules without question due to fear of punishment.

Which accurately describes hospice care? 1. A resident's temporary or permanent home, where the surroundings have been made as homelike as possible 2. Offers an attractive long-term care setting with an environment akin to the client's home, which offers the client greater autonomy 3. Service that provides short-term relief for people providing home care to an ill, disabled, or frail older adult 4. System of family-centered care that allows clients to remain at home in comfort while easing the pain of terminal illness

4. System of family-centered care that allows clients to remain at home in comfort while easing the pain of terminal illness *Hospice care is a system of family-centered care that allows clients to remain at home in comfort while easing the pain of terminal illness. A nursing center is a resident's temporary or permanent home, where the surroundings are made as homelike as possible. Assisted living offers an attractive long-term care setting with an environment that is like the client's home and offers the client greater autonomy. Respite care is a service that provides short-term relief for people providing home care to an ill, disable, or frail older adult.

By asking her to describe her concerns more fully

A new mother with class II heart disease tells a nurse that she is afraid that her heart condition will prevent her from caring for her baby and her home when she is discharged. How should the nurse respond?

Thin upper lip Small upturned nose Smooth vertical ridge in the upper lip

A new mother's laboratory results indicate the presence of cocaine and alcohol. Which craniofacial characteristic indicates to the nurse that the newborn has fetal alcohol syndrome (FAS)? (Select all that apply.)

Decreased blood pH

A preterm infant with respiratory distress syndrome (RDS) has blood drawn for an arterial blood gas analysis. What test result should the nurse anticipate for this infant?

Cephalhematoma

During the discharge examination of a 2-day-old newborn, the nurse observes an edematous area confined to the right side of the scalp. How should the nurse document this condition?

What is a basic concept associated with rehabilitation that the nurse should consider when formulating discharge plans for clients?

Immediate or potential rehabilitation needs are exhibited by clients with health problems

The Institute of Medicine (IOM) identified five interrelated competencies for all health care workers in the twenty-first century. What should the nurse do to provide patient-centered care?

Share decision-making and management.

Which statement made by a parent alerts the nurse to the need for additional education about poison prevention? a. I keep the poison control center phone number easily accessible. b. All medication is kept out of reach in a locked cabinet. c. I keep a bottle of syrup of ipecac handy. d. Our garden is free from marigolds.

c

Which nursing interventions will be implemented for the mother of a 10-month-old infant with nonorganic failure to thrive? a. Pointing out errors that the nurse observes when the mother is caring for the infant b. Discussing negative characteristics of the infant with the mother c. Having the nurse provide as much of the infants care as possible d. Teaching the mother about the developmental milestones to expect in the next few months

d

A 16-year-old girl has been admitted to the pediatric eating disorders unit with a diagnosis of anorexia and is undergoing behavioral therapy. Unit privileges are based on weight gain and have been explained to the client. What is the most appropriate intervention for the nurse to use when taking the lunch tray to the client's room? Setting the tray down and saying nothing Reminding the client that eating will be rewarded Commenting on the client's thinness and need to gain weight Threatening the client that if she doesn't eat she won't gain any privileges

The client uses eating/weight gain as a means of controlling the environment. The client has been told the rules of the unit and must make the personal decision to try to win privileges. The nurse needs to take the focus away from eating. The client knows that gaining weight will be rewarded and does not need reminders. The client is used to everyone commenting on her weight. Although the client appears thin to others, the client's self-perception is that she needs to lose a little bit of weight. Threats should not be used in any circumstance.

Keeping the infant in a warm environment

What nursing care is most important for a newborn with respiratory distress syndrome (RDS)?

Pneumonia Preterm birth Conjunctivitis

A nurse is caring for a new mother who has a chlamydial infection. Which complications are associated with chlamydial infections in neonates? (Select all that apply.)

Supporting the parents

A nurse is caring for a newborn with a cephalohematoma. What is the priority nursing action?

Breaks down the bilirubin into a conjugated form

A nurse is caring for a preterm neonate with physiological jaundice who requires phototherapy. What is the action of this therapy?

Having sex with many partners

A nurse is caring for the newborn of a drug-addicted mother with suspected cytomegalovirus disease. What does the nurse suspect was the cause of the disease?

Flaring nares

A nurse is observing a newborn of 33 weeks' gestation. Which sign alerts the nurse to notify the health care provider?

Irregular, abdominal, 30 to 60/min

A nurse is observing a newborn's respiratory rate. What clinical findings indicate that the rate is within the expected range?

Sneezing Hyperactivity High-pitched cry

A nurse is observing the newborn of a known opioid user for signs of withdrawal. What clinical manifestations does the nurse expect to identify? (Select all that apply.)

An audible click on abduction

A nurse is performing the Ortolani test on a newborn. Which finding indicates a positive result?

A client with asthma and depression is admitted to the hospital. Which tasks delegated by the delegator would indicate the task is applicable for the registered nurse?

Teach the client how to use a metered-dose inhaler

Which delivery system was developed to provide care through others as an adaptation to functional nursing?

Team

Warm the environment

While observing a newborn, the nurse notes that the skin is mottled. What should the nurse do first?

It contains exposed tissue and blood

While showing a new mother how to care for her infant's umbilical cord stump, the nurse explains that the stump is a potential source of infection because:

A registered nurse teaches a nursing student about routines followed during a physical examination to help ensure that important findings are not missed. Which statement by the nursing student indicates ineffective learning?

"I'll perform painful procedures at the beginning of the examination."

Which statement by the mother of a 5-month-old infant indicates effective learning about proper nutrition for the infant's growth and development? 1 "I will breastfeed my child." 2 "I will give my child whole cow's milk." 3 "I will give my child adequate fruit juice." 4 "I will give my child iron-fortified cereals."

1 "I will breastfeed my child." Breastfeeding is recommended for infant nutrition because breast milk contains essential nutrients of proteins, fats, carbohydrates, and immunoglobulins. Giving cow's milk to an infant may lead to internal bleeding, anemia, and an increased incidence of allergies. Fruit juices should be avoided because they do not provide sufficient calories during this period. Iron-fortified cereals should be given to infants after 6 months of age because infants younger than 6 months of age are not sufficiently mature to digest solid foods.

n which phase does a child develop the sense of object permanence according to Jean Piaget? 1 Sensorimotor 2 Preoperational 3 Formal operations 4 Concrete operations

1 Sensorimotor During the sensorimotor period, the child understands that objects continue to exist even when they cannot be seen, heard, or touched. This is called object permanence. During the preoperational phase, children learn to think with the use of symbols and mental images. Egocentricity is observed in the formal operations period. Children are able to perform mental operations during the concrete operations period.

A 9-year-old child has a fractured tibia, and a full leg cast is applied. Which assessment findings should the nurse immediately report to the health care provider? Select all that apply. 1. Inability to move the toes 2.Increased urine output 3.Pedal pulse of 90 beats/min 4.Tingling sensation in the foot 5.Fiberglass cast that is damp after 4 hours

1, 4 & 5 A cast is not flexible and can inhibit circulation. Cold toes, loss of sensation in toes, pain, and inability to move the toes should be reported immediately. A tingling sensation in the foot may indicate excessive pressure on the nerves and circulatory system in the casted extremity. A fiberglass cast dries within minutes; if it remains damp, it should be reported before 4 hours have elapsed. Increased urine output is not significant; it may be related to increased fluid intake. The expected pulse rate for a 9-year-old child ranges from 70 to 110 beats/min.

An 8-year-old child is admitted to the pediatric unit with nephrotic syndrome. What measures should the nurse expect to include in the plan of care for this child? Select all that apply 1.Providing symptomatic care 2.Maintaining bedrest 3.Administering antibiotics 4.Eliminating high-sodium foods 5.Monitoring response to steroids

1, 4 & 5 Bedrest for children with nephrotic syndrome is generally no longer ordered. When there is gross edema, children usually prefer to remain in bed to conserve energy, but there are no ill effects of ambulating if they wish to do so. Nephrotic syndrome is a noninfectious disorder; however, these children are prone to infection, and if they contract an infection it is treated accordingly. Examples of symptomatic care are treating azotemia with a low-protein diet; encouraging bedrest if there is gross edema; restricting fluids if there is oliguria; and treating infection if it should occur. Foods that are high in sodium are restricted when there is gross edema; although restricting foods that are high in sodium does not lessen the edema, it seems to prevent it from worsening. A steroid is given to children with nephrotic syndrome because of its antiinflammatory properties. It is essential that the nurse monitor the child's response to steroids to determine the medication's effectiveness.

A healthcare facility is using the "plan, do, study, act (PDSA)" cycle model for performing a quality improvement. Arrange the order in which quality improvement takes place based on this model.

1. Review available data. 2. Choose the appropriate intervention. 3. Evaluate the outcomes. 4. Incorporate new practices in daily performance

Which antipyretic medication may cause Reye syndrome in children? 1. Aspirin 2. Naproxen 3. Ibuprofen 4. Dantrolene

1. Aspirin * Aspirin increases the risk of swelling in the brain and liver, which are the main symptoms of Reye Syndrome in children. Aspirin is not recommended in children. Medications such a naproxen and ibuprofen do not include swelling in the brain and liver; therefore, these medications may not cause Reye syndrome. Dantrolene does not include swelling in the brain and liver; instead, it decreases calcium levels during malignant hyperthermia conditions.

A child with recently diagnosed idiopathic scoliosis has a mild structural curve. The child's mother asks whether the problem can be corrected with exercise. What should the nurse tell the mother concerning an exercise program? 1.Exercise is used in conjunction with a brace. 2.Exercise can be used if the child appears highly motivated. 3.Exercise might exaggerate the curvature if the curve is severe. 4.Exercise is needed to correct the curvature without the need for a brace.

1. Exercise is used in conjunction with a brace. An exercise program and a brace are the treatments of choice for mild structural scoliosis. Although compliance will affect the ultimate outcome of treatment, exercises alone are not helpful in this type of scoliosis. Exercises are to be encouraged, regardless of the type or extent of scoliosis. Exercises alone are used only with postural-related, not structural-related, scoliosis.

A client presents with chief complaints of unexplained weight gain and back pain from a compression fracture of the vertebrae. On assessment, there is truncal obesity with excessively thin extremities, a moon-shaped face, a buffalo hump, thin hair, and adult acne. The symptoms described are suggestive of which disease? Addison disease Cushing disease Multiple sclerosis Kaposi sarcoma

2

Which condition would the nurse document to describe a client presenting with the loss of the ability to taste after cancer treatment has affected the client's ability to eat food? 1 Mucositis 2 Dysgeusia 3 Dysphagia 4 Xerostomia

2

Which statement by a mother indicates the need for additional teaching about safety guidelines for infants and toddlers? 1 "I will use plastic eating and drinking utensils." 2 "I will give my child hard candies for chewing." 3 "I will not allow my child to chew on old furniture." 4 "I will turn pot handles toward the back of the stove."

2 "I will give my child hard candies for chewing." Hard candies should not be given to infants and toddlers because they can be easily aspirated, which will result in choking. Therefore, the nurse should suggest that the mother avoid giving hard candies to young children. The nurse should suggest that the mother use plastic eating and drinking utensils for young children because glass and ceramic utensils may break and lead to injury. Old furniture may contain lead paints, which are toxic. Therefore, children should not be allowed to chew on them. Some children, out of curiosity, may grab pot handles, which can cause burns. Therefore, the nurse should advise the mother to turn pot handles toward the back of the stove for the safety of young children.

The community nurse is assessing an elderly client who lives alone at home. The nurse finds that the client refrains from physical activity for fear of falling when walking. Which interventions by the nurse are most beneficial to promote a healthy lifestyle? Select all that apply. 1 Instruct the client to apply bed side rails. 2 Encourage the client to wear nonskid shoes. 3 Suggest that the client use an assistive device. 4 Ask the client to install hand rails in the bathroom. 5 Help the client rearrange furniture in the house.

2 Encourage the client to wear nonskid shoes. 3 Suggest that the client use an assistive device. 5 Help the client rearrange furniture in the house.

Which is the definition of a tort? 1. The application of force to the body of another by a reasonable individual 2. An illegality committed by one person against the property or person of another 3. Doing something that a reasonable person under ordinary circumstances would not do 4. An illegality committed against the public and punishable by the law through the courts

2. An illegality committed by one person against the property or person of another * An individual is held legally responsible for actions committed against another individual or an individuals property. The application for force to the body of another is battery, which involves physical harm. Doing something that a reasonable person under ordinary circumstances would not do is the definition of negligence. An illegality committed against the public and punishable by the law through the courts is the definition of a crime.

The nurse in the pediatric clinic is reviewing the health history of a 10-year-old girl with a diagnosis of juvenile idiopathic arthritis (JIA). Currently the child is experiencing recurrent pain and swelling of the joints, particularly her knees and ankles. What organ is commonly affected in children with this disorder? 1.Ears 2.Eyes 3.Liver 4.Brain

2. Eyes Juvenile idiopathic arthritis can cause inflammation of the iris and ciliary body of the eyes, which may lead to blindness. The ears are not affected. The liver may become enlarged, but this does not occur as frequently as visual problems do. The brain is not affected.

Which is an appropriate action for the registered nurse regarding assisted suicide? 1. Nurses may have an open attitude towards the clients end of life 2. Nurses participate in assisted suicide violates the code of ethics 3. Nurses may listen to the clients expression of fear and attempt to control the clients pain 4. Nurses can participate in assisted suicide only if the individual could make an oral and written request

2. Nurses participate in assisted suicide violates the code of ethics * According to the American Nurses Association (ANA), the nurses participation in assisted suicide would violate there code of ethics. According to the American Association of Colleges of Nurses (AACN) and the International Council or Nurses, the nurse may have an open attitude towards the clients end of life. According to the AACN and the International Council of Nurses, nurses may listen to the clients expressions of fear and attempt to control the clients pain. According to the Oregon Death with Dignity Act (1994), the primary health care provider in the state of Oregon can participate in assisted suicide only if an individual with a terminal disease makes an oral and written request to end his or her life in a humane and dignified manner.

Which is the subset of clinical health care informatics? 1. Clinical informatics 2. Nursing informatics (NI) 3. Public health informatics 4. Clinical research informatics

2. Nursing informatics (NI) * NI is the subset of clinical health care informatics. This is a specialty that integrates nursing science, computer science, and information science to manage and communicate data, information, knowledge, and wisdom in nursing practice. Clinical health care informatics is a subdomain of clinical informatics. Public health informatics is one of the major domains of informatics that uses computer science and technology to improve public health. Clinical research informatics is a subdomain of clinical informatics.

A newborn who was delivered with the assistance of forceps sustains an injury that results in facial paralysis. What would the nurse state to the mother? 1 The baby will have this condition for life. 2 The newborn may need intensive physiotherapy. 3 The condition usually subsides on its own in a few days. 4 The newborn should not be allowed to cry because it can cause pain.

3 The condition usually subsides on its own in a few days. A difficult delivery performed with forceps may result in facial paralysis, which may manifest as asymmetrical movements of the face, an inability to close the eyelid, and drooping of the corner of the mouth. This condition is self-limiting and may subside in few days. Physiotherapy is not indicated for the treatment of this condition. The parents should be informed that this condition is not painful.

A 2-year-old toddler is admitted to the pediatric unit with a diagnosis of bacterial meningitis. What is the most important safety measure for the nurse to institute immediately after the child has a seizure? 1.Monitoring the child's vital signs 2.Padding the side rails of the toddler's crib 3.Placing the child in the side-lying position 4.Bringing suction equipment to the bedside

3. Placing the child in the side-lying position The side-lying position promotes a patent airway; the tongue can move away from the back of the pharynx and saliva can flow out of the mouth by gravity. Although monitoring of vital signs is important, a patent airway is the priority. Suctioning may be unnecessary; the child should not be left alone while equipment is obtained. The crib sides should have been padded as a part of seizure precautions before the seizure. If the seizure was unexpected and seizure precautions were not previously instituted, they should be instituted after the immediate respiratory and safety needs of the toddler have been met.

Which site is best used to assess a client for jaundice? 1. Skin 2. Palm 3. Sclera 4. Conjunctiva

3. Sclera * The sclera is the best site to inspect for jaundice. Because the skin may become pale due to anemia or jaundice, a skin inspection is not recommended. The palms and conjunctive are inspected to assess pallor.

Which variable is an example of an external variable? 1. Spiritual factors 2. Developmental issues 3. Socioeconomic factors 4. Perception of functioning

3. Socioeconomic factors *Socioeconomic factors are considered to be external variable. Spiritual factors, development issues, and the perception of functioning are internal variables.

After surgery a client is to receive an antibiotic by intravenous (IV) piggyback in 50 mL of a diluent. The piggyback is to infuse in 20 minutes. The drop factor of the IV set is 15 gtts/mL. The nurse should set the piggyback to flow at how many gtts/min? Record your answer using a whole number. __________ gtts/min

38

When does the Babinski reflex disappear? 1 By 6 months 2 By 4 months 3 By 3 months 4 After 12 months

4 After 12 months The Babinski reflex disappears after 1 year or 12 months. The Perez reflex disappears by 6 months. The extrusion reflex and the Moro reflex may disappear by 4 months. The rooting reflex may disappear by 3 months.

The nurse teaches a parent about managing nocturnal enuresis for a 7-year-old child. Which action by the parent would be helpful in managing the child? 1 Making the child wear a diaper 2 Limiting fiber in the child's diet 3 Giving the child cranberry juice in the evening 4 Waking up the child at night to use the bathroom

4 Waking up the child at night to use the bathroom Wetting the bed at night, called nocturnal enuresis, may be managed by waking up the child at night to use the bathroom, relieving the bladder. The child should wear regular sleepwear, and parents should avoid use of diapers or pull-ups. Constipation may contribute to enuresis, so dietary fiber should be increased. Fruit juices and beverages are high in water content, and therefore result in an urge to urinate.

A nurse is assessing a client's degree of edema and finds 8 mm of depth. How does the nurse document this condition?

4+

A client is dying. Hesitatingly, his wife says to the nurse, "I'd like to tell him how much I love him, but I don't want to upset him." Which is the correct response by the nurse? 1. "You must keep up a strong appearance for him." 2. "I think he'd have difficulty dealing with that now." 3. "Don't you think he knows that without you telling him?" 4. "You should share your feelings with him while you can."

4. "You should share your feelings with him while you can." * It is difficult to work through loss; however, encouraging the sharing of feeling helps both parties feel better about having to let go. The response, "You must keep up a strong appearance for him" impedes the work of acceptance of one's finality and the use of the remaining time to the best advantage. There is not evidence to suggest that the client cannot cope with these emotions; the response, "I think he'd have difficulty dealing with that now" denies that this is a time for closeness and honestly. The response, "Don't you think he knows that without you telling him?" is demeaning, closes off communication, and does not foster the expression of feelings.

The nurse notes that a client has mild hypothermia based on which body temperature? 1. 29 C 2. 30 C 3. 33 C 4. 35 C

4. 35 C * Hypothermia occurs when the body temperature falls below 36.2 C. based on the severity, it is classified as mild, moderate, and severe. Mild hypothermia refers to a body temperature of 34 C to 36 C (93.2 - 96.8 F). In this case, the client's body temperature is 35 C, which indicates mild hypothermia. Moderate hypothermia refers to a body temperature below 30 C (86 F). The client does not have severe hypothermia; therefore, the client does not have a body temperature of 29 C. The client does not have moderate hypothermia; therefore, the client does not have a body temperature of 30 C or 33 C.

Which potential health problem would the nurse include in the young adults discharge teaching? 1. Kidney disfunction 2. Cardiovascular diseases 3. Eye problems, such as glaucoma 4. Accidents, including their prevention

4. Accidents, including their prevention *Accidents are common during young adulthood because of immature judgement and impulsivity associated with this stage of development. Kidney dysfunction is not a problem specific to any one stage of growth. Cardiovascular disease is a common health problem in middle adulthood. Glaucoma is a common health problem in older adults.

Refusing to followed the prescribed treatment regimen, a client plans to leave the hospital against medical advise. Which is it important for the nurse to inform the client of? 1. The client is acting irresponsibly 2. This action violates the hospital policy 3. The client must obtain a new primary health care provider for future medical needs 4. The client must accept full responsibility for possible undesirable outcomes

4. The client must accept full responsibility for possible undesirable outcomes * The client has the right to self-determination, which includes refusing medical treatment. However, if the client does so, he or she must accept full responsibility for the illness and possible injury or undesirable outcomes. Health care professionals have a responsibility to inform the client and, if possible, have the client sign an informed waiver or a leaving against medical advice document. Acting irresponsibly is a subjective assumption. The client may be violating the hospital policy; however, if the client is deemed competent, he or she has the right to refuse treatment. Leaving against medical advice does not mean that the current primary health care provider will refuse to provide care to the client in the future.

While assessing a client for the dorsalis pedis pulse, the nurse documents the reading at 1+. Which can be inferred from this finding? 1. There is absence of a pulse 2. The pulse strength is normal 3. The pulse strength is bounding 4. The pulse strength is barely palpable

4. The pulse strength is barely palpable * A pulse strength of 1+ indicates a diminished or barely palpable pulse and requires immediate intervention. Absence of pulse is documented as 0. Normal pulse strength is documented at 2+. If the pulse strength is bounding, then it is documented as 4+.

A nurse is assessing clients who are to be given the smallpox vaccination. Which client should the nurse remove from the immunization line for medical counseling?

45-year-old woman with breast cancer

A 2.5-year-old child is admitted for treatment of injuries supposedly sustained in a fall down a flight of stairs. Child abuse is suspected. What statements might the nurse expect from a parent who engages in child abuse? Select all that apply. "Kids have to learn to be careful on the stairs." "Every time I turn around the kid is falling over something." "This child tends to be adventurous and doesn't understand about getting hurt on the stairs." "I can't understand it. This child didn't have a problem using the stairs without my help before this." "I try to keep an eye on my child, but little kids are always on the go and I just can't keep running after the kid."

Abusive parents often have a poor understanding of the expected growth and development of children and tend to blame the child. Toddlers generally need supervision and some assistance when climbing stairs, but abusive parents have little understanding of toddlers' abilities. Although "Kids have to learn to be careful on the stairs" is a true statement about toddlers, it is an unlikely response from an abusive parent because these people do not have an understanding of children's needs in relation to growth and development. "This child tends to be adventurous and doesn't understand about getting hurt on the stairs" is an unlikely response from an abusive parent because these people do not have an understanding of children's needs in relation to growth and development. Although "I try to keep an eye on my child, but little kids are always on the go and I just can't keep running after the kid" is a true statement about toddlers, it is an unlikely response from an abusive parent because these people usually do not have an understanding of children's needs in relation to growth and development.

Which task can be delegated to the licensed vocational nurse (LVN)? Select all that apply.

Administering oral hypoglycemic agents administering intramuscular injections

A client with a recent history of sinusitis develops meningitis and demonstrates a positive Brudzinski sign. The priority nursing care is:

Administering prescribed antibiotics

Metabolic acidosis

After a newborn has skin-to-skin contact with the mother, a nurse places the newborn under a radiant warmer. What complication is the nurse attempting to prevent?

"Newborns are deficient in vitamin K. This treatment will protect your baby from bleeding."

After the birth of her daughter, a mother tells the nurse, "I was told that my baby has to have an injection of vitamin K. She's so small to be getting a shot. Why does she have to have it?" How should the nurse respond?

How many deciduous teeth would be present in a 10-month-old infant? Record your answer as a whole number. ________

Age of child in months minus 6 = Number of deciduous teeth. Therefore, the number of deciduous teeth is 10 - 6 = 4.

Bulging fontanels

An infant has surgery for repair of a myelomeningocele. For which early sign of impending hydrocephalus should the nurse monitor the infant?

A client is placed on a restricted diet. What is the best communication technique for the nurse to use when beginning to teach the client about the diet? Asking about what type of foods the client usually eats Telling the client that the diet must be followed exactly as written Telling the client that the intake of foods on the list must be limited Asking about what the client knows about the diet that was prescribed

Asking about what the client knows about the prescribed diet may validate the client's understanding; the response may indicate the need for further teaching or that the client understands; understanding and accepting the need for restrictions will increase adherence to the diet. Assessing the client's food preferences and teaching about diets follow an assessment of the client's understanding about the need for a specific diet; the client must understand the need for and the benefits of the diet before there is a readiness for learning. Telling the client that the diet must be followed exactly as written and telling the client that the intake of foods on the list must be limited are authoritarian and should be avoided.

Suctioning the mouth

At 10 hours of age a newborn has a large amount of mucus in the nasopharynx and becomes cyanotic. What is the nurse's initial action?

Wrinkled skin Long nails

At 42 weeks' gestation a client gives birth to an 8-lb 5-oz newborn. On examining the infant, what does the nurse expect to observe? (Select all that apply.)

Which clinical condition will result in changes in the integrity of the arterial walls and small blood vessels?

Atherosclerosis

According to the Model for Differentiated Nursing Practice, which entry-level nurse is most prepared for prioritization of client care?

Bachelor's degree registered nurse

A client has a Mantoux test as part of a yearly physical examination. The area of induration is 10 mm within 48 hours after having the test. The nurse concludes that this response indicates that the client has:

Been exposed to the tubercle bacillus

A client who underwent a physical examination reports itching after 2 days. Which condition should the nurse suspect?

Contact dermatitis

A new mother who is unable to breastfeed her newborn expresses breast milk to feed the baby. Which actions of the mother should be corrected by the nurse? Select all that apply. A Adding water to dilute the expressed milk B Storing expressed milk at room temperature C Expressing milk by hand or with breast pump D Storing expressed milk in the refrigerator for 5 days E Adding honey to expressed milk to improve taste

Correct A Adding water to dilute the expressed milk Correct B Storing expressed milk at room temperature Correct E Adding honey to expressed milk to improve taste

A 13-year-old child states, "I don't know if I want to go to college or start working after high school." Which stage of psychosocial development is indicated by this child's uncertainty? 1 Intimacy vs. isolation 2 Ego integrity vs. despair 3 Identity vs. role confusion 4 Generativity vs. stagnation

Correct3 Identity vs. role confusion This child's statement indicates a struggle to establish a sense of identity, which is characteristic of the identity vs. role confusion stage. The intimacy vs. isolation stage is characterized by establishing intimate bonds of love and friendship. Looking back over one's life and accepting its meaning are observed in the ego integrity vs. despair stage of psychosocial development. The generativity vs. stagnation stage is seen in middle adulthood, where fulfilling life goals that involve family, career, and society plays an important role.

Which theory describes the phenomenon of grief or caring?

Descriptive theories

A newly immigrated older Chinese adult is brought to a mental health clinic when family members become concerned that their parent is depressed. In an attempt to conduct a culturally competent assessment interview, the nurse asks certain questions. Which questions does the nurse ask? Select all that apply. "What brought you here for treatment today?" "What do you believe is the cause of your depression?" "Does religion have a role in your perception of health and wellness?" "Do you have insurance that includes coverage of mental health issues?" "Have you ever sought treatment for a mental health problem previously?"

Determining the client's perception of the problem is an appropriate question that allows cultural factors to be included. Encouraging the client to discuss the problems will facilitate a clearer understanding of the factors involved. Religion often plays a significant role in a client's view of health, wellness, and recovery. Knowing whether a client has ever undergone treatment for a mental health problem reveals mental health history and how previous issues were addressed. Insurance coverage is not pertinent to the issue and is an inappropriate topic of questioning by the nurse.

A visitor says to the nurse, "Can I read my client's progress record? I am the sponsor from an alcohol recovery program." How should the nurse respond?

Do not allow the sponsor to review the record

Which degree does The American Association of Colleges of Nursing (AACN) recommend as terminal practice degree for all advanced practice registered nurses (APRNs)?

Doctor of Nursing Practice (DNP)

Suctioning the airway

During a vertex vaginal birth the nurse notes meconium-stained amniotic fluid. What is the priority nursing intervention for the newborn?

Which theorist suggested that the goal of nursing is to use communication to help clients reestablish a positive adaptations to their environments?

King

A nurse is assessing an 89-year-old client with a history of severe congenital spinal deformity. Which condition would most likely describe the nurse's finding?

Kyphosis

The parent of a 4-year-old client states, "My child gets so upset when I ask her to stop talking so that I can answer a work phone call. I don't understand why this happens." Which response by the nurse is most appropriate? "School is a source of stress for many children at this age." "Worrying is a source of stress for many children at this age." "Attention is a source of stress for many children at this age." "Belongings are a source of stress for many children at this age."

Lack of attention is often a stressor for the 4-year-old preschool-age client; therefore, the nurse should provide education about this stressor to the parent. The child's reaction is not due to the stressors of school, worrying, or belongings

The nursing manager is preparing a schedule for delegating appropriate tasks to different health care team members. Which health care team member can be delegated the task of administering oral medications? Select all that apply.

Licensed practical nurse Licensed vocational nurse

A client is concerned about contracting malaria while visiting relatives in Southeast Asia. What should the nurse teach the client to avoid to best prevent malaria?

Mosquito bites

A pregnant woman complains, "I have had morning sickness for one week, which caused me to eat less. I am experiencing irritability because of weakness." The nurse taught the client relaxation and distraction techniques to overcome this problem. During a follow-up visit, the nurse evaluates the client's response and determines if the condition is resolved. Which model is the nurse using in this situation?

Neuman systems model

A registered nurse is teaching a nursing student about Nightingale's theory of nursing. Which statements have been correctly stated by the nursing student as a result of the teaching? Select all that apply.

Nightingale's theory states that the focus of nursing is caring through the environment. Nightingale's theory is oriented towards providing fresh air, light, warmth, cleanliness, quiet, and adequate nutrition. Nightingale's theory focuses on helping the client deal with the symptoms and changes in function related to an illness.

Which are the core roles for an advanced practice registered nurse (APRN)? Select all that apply.

Nurse practitioner Clinical nurse specialist (CNS) Certified nurse midwife (CNM) Certified registered nurse anesthetist (CRNA)

What are physiologic symptoms assessed in a client with sleep deprivation? Select all that apply.

Ptosis may result from a loss of elasticity of the eyelids, which is a physiologic symptom of sleep deprivation. Decreased auditory alertness and blurred vision are also physiologic symptoms of sleep deprivation. Agitation, hyperactivity, confusion, disorientation, and increased sensitivity to pain are psychologic symptoms of sleep deprivation.

What should the nurse do when a client with the diagnosis of schizophrenia talks about being controlled by others? Express disbelief about the client's delusion. Divert the client's attention to unit activities. React to the feeling tone of the client's delusion. Respond to the verbal content of the client's delusion.

Reacting to the feeling tone of the client's delusion helps the client explore underlying feelings and allows the client to see the message that the verbalizations are communicating. Expressing disbelief about the client's delusion denies the client's feelings rather than accepting and working with them. Attempting to divert the client rather than accepting and working with the client denies the client's feelings. Responding to the verbal content of the client's delusion focuses on the delusion itself rather than on the feeling that is causing the delusion.

The nurse is assessing a client who is undergoing chemotherapy. The nurse notes that the client is using a scarf to cover the head. The nurse asks the client about coping with the altered body image. Which functional pattern would the assessment include? 1. Value-belief pattern 2. Role-relationship pattern 3. Cognitive-perceptual pattern 4. Self-perception-self-tolerance pattern

Self-perception-self-tolerance pattern * The nurse is applying Gordon's self-perception-self-tolerance pattern describes the client's self-worth, emotional patterns, and body image. The value belief pattern describes patterns of values, beliefs, spiritual practices, and goals that guide the clients choices or decisions. The role-relationship pattern describes patterns of role engagements and relationships. the cognitive-perceptual pattern describes sensory-perceptual patterns, language adequacy, memory, and decision-making ability.

A new toy is shown to a baby, and after his or her attention is drawn, the parent hides the toy under the bed. Then the baby tries to find the toy, which is hidden. Which stage of cognitive development does the child belong, according to Jean Piaget?

Sensorimotor

SMART goals

Specific, Measurable, Attainable, Realistic, Timely

Which activity of the registered nurse is included in supervision? Performing the given task Allocating a portion of the work Providing information to the delegator Providing guidance and oversight in delegating a task

Supervision is defined as "provision of guidance and oversight of a delegated nursing task." It includes open lines of communication between the delegator and the delegatee to provide guidance. Responsibility is the activity of performing a given task. Delegation is the activity of allocating a portion of work. Authority is the activity of providing information to the delegator.

Verifying oxygen saturation frequently to adjust flow on the basis of need

Supplemental oxygen is ordered for a preterm neonate with respiratory distress syndrome (RDS). What action does the nurse take to reduce the possibility of retinopathy of prematurity?

The nurse cares for a client who develops pyrexia three days after surgery. The nurse should monitor the client for which signs and symptoms commonly associated with pyrexia? (Select all that apply.)

Tachypnea Increased pulse rate

Which of the following is a description of the percussion technique?

Tapping the skin with the fingertips to vibrate underlying tissues

Arrange the events of communication throughout the nursing process in chronological order. 1. Assessing the medical records and diagnostic tests 2.Documenting expected outcomes Incorrect 3.Performing verbal, visual, auditory, and tactile health teaching activities 4.Intrapersonal analysis of the assessment findings 5.Identifying the factors affecting the outcomes

The first step of communication throughout the nursing process is assessment, which involves assessing medical records and diagnostic tests. The second step is nursing diagnosis, which involves the intrapersonal analysis of assessment findings. The third step is planning, which involves the documentation of expected outcomes. The fourth step is implementation, which involves performing verbal, visual, auditory, and tactile health teaching activities. The final step is evaluation, which involves identifying the factors affecting the outcomes.

What is the duty of a nurse while caring for a client?

The nurse should determine the client's care preferences.

After a week on the mental health unit, a client with the diagnosis of paranoid schizophrenia continues to say, "They're trying to kill me. They all are." What is the best response by the nurse? "We're here to protect you." "No one wants to hurt anyone." "You're having very frightening thoughts." "Tell me more about their wanting to kill you."

The observation that the client is experiencing frightening thoughts is a reflection of the client's feelings; it leaves the line of communication open. Telling the client that the staff is there to protect the client does not provide security, because the client may believe that the nurse is one of the people plotting. Telling the client that no one wants to hurt anyone discounts the client's thoughts and may increase the agitation. Asking the client to detail the plot supports the client's delusion.

Most important is the institution of a corrective formula soon after birth.

The parents of a newborn with phenylketonuria (PKU) ask a nurse how to prevent future problems. What must the nurse consider before responding?

What description of a childs stool characteristic leads the nurse to suspect intussusception? a. Currant jelly b. Black and tarry c. Green liquid d. Greasy and foul-smelling

a

Evidence-informed nursing uses a variety of sources to support nursing practice. Which are sources of evidence-informed practice? Select all that apply.

Theory Research Clinical expertise

Immature liver function

Three days after birth, a breastfeeding newborn becomes jaundiced. The parents bring the infant to the clinic and blood is drawn for an indirect serum bilirubin determination, which reveals a concentration of 12 mg/dL. The nurse explains that what the infant has is physiological jaundice, a benign condition, caused by:

A nurse is caring for a client with acquired immunodeficiency syndrome (AIDS). What precautions should the nurse take when caring for this client?

Use standard precautions.

A child has been diagnosed with ascariasis (roundworm). Which statement made by her mother that may suggest a cause for her condition? a. Ive been airing out the house on these nice breezy days. b. My child often goes out to the garden and pulls up a carrot to eat. c. She runs barefoot so much I have to wash her feet at least twice a day. d. We just remodeled our bathroom at home.

b

A child is brought to the emergency department because he ingested an unknown quantity of acetaminophen (Tylenol). What does the nurse expect this child to receive following gastric lavage? a. Activated charcoal b. N-acetylcysteine c. Vitamin K d. Syrup of ipecac

b

A frightened mother calls the pediatricians office because her child swallowed dishwashing detergent. What is the most appropriate action? a. Induce vomiting by giving the child syrup of ipecac. b. Take the child to the local emergency department. c. Give the child activated charcoal mixed with juice. d. Give the child milk to soothe affected mucous membranes

b

What would the nurse expect to find in a child admitted to the hospital for nonorganic failure to thrive? a. Cry to be picked up b. Be limp like a rag doll c. Be responsive to cuddling d. Weigh in the 10th percentile for age

b

Which finding in a newborn is suggestive of tracheoesophageal fistula? a. Failure to pass meconium in 24 hours b. Choking on the first feeding c. Palpable mass in the sternal area d. Visible peristalsis across abdomen

b

A mother reports that her 2-year-old child experiences constipation frequently. Which food would the nurse recommend to include in the childs diet? a. Cooked vegetables b. Pretzels c. Whole-grain cereal d. Yogurt

c


Related study sets

Rheumatic Disorders (RA, SLE, Gout, Fibromyalgia etc.)

View Set

RNSG-1105-Module 2 Infection Prevention and control

View Set

Thinking & Intelligence and Memory Pretest (4/11 - 4/24)

View Set